Bank of Baroda PO Examination Held on 17-05-2017 General Awareness Question Paper With Answer Key

Bank of Baroda PO Examination Held on 17-05-2017 General Awareness
Bank of Baroda PO Examination Held on 17-05-2017 General Awareness Question Paper With Answer Key

Bank of Baroda PO Examination Held on 17-05-2017

General Awareness

 

1. ‘MUDRA Bank’ is setup for the development of micro units and to provide funding to the none-corporate small business sector. The alphabet ‘D’ in the abbreviation ‘MUDRA’ stands for

(A)  Development

(B)  Disintermediation

(C)  Demonetization

(D)  Delhi

(E)  Discipline

Answer: (A)

2. The Airtel Payment Bank (APB) is the joint venture of Bharti Airtel Limited and-

(A)  Union Bank of India

(B)  State Bank of India

(C)  Kotak Mahindra Bank

(D)  HDFC

(E)  ICICI

Answer: (C)

3. ‘Malyalam’ is a language predominantly spoken in the Indian and State of-

(A)  Andhra Pradesh

(B)  Karnataka

(C)  Telangana

(D)  Tamil Nadu

(E)  Kerala

Answer: (E)

4. The ‘Deodhar Trophy’ is associated with the game of-

(A)  Hockey

(B)  Tennis

(C)  Badminton

(D)  Basketball

(E)  Cricket

Answer: (E)

5. The ‘Financial Intelligence Unit (FIU)’ is the national agency mandated to collect, analyzed and disseminate CTRs and STRs. The alphabet ‘S’ in the abbreviation ‘STR’ stands for-

(A)  Serious

(B)  Scandafous

(C)  Statistical

(D)  Structured

(E)  Suspicious

Answer: (E)

6. National Coach Curbax Singh Sandhu is associated with-

(A)  Women’s hockey

(B)  Women’s boxing

(C)  Women’s cricket

(D)  Women’s basketball

(E)  Women’s Football

Answer: (B)

7. The Reserve Bank of India has recently come out with a revised PCA framework on which banks would be monitored. The alphabet ‘P’ in the abbreviation ‘PCA’ stands for-

(A)  Provision

(B)  Package

(C)  Performance

(D)  Prompt

(E)  Print

Answer: (D)

8. The joint military exercise ‘Sagarmatha Friendship 2017’ focusing on counter-terrorism was recently (April 2017) held by the armies of China and-

(A)  Bhutan

(B)  Nepal

(C)  Pakistan

(D)  Sri Lanka

(E)  Myanmar

Answer: (B)

19. ‘AEPS’ is a payment service offered by the NPCI to banks, financial institutions using Aadhaar number and UIDAI authentication. The alphabet ‘E’ in the abbreviation ‘AEPS’ stands for-

(A)  Econometric

(B)  Equiry

(C)  Enabled

(D)  Emergency

(E)  Electronic

Answer: (C)

10. Which of the following is not a part of Digital banking campaign?

(A)  Internet for all

(B)  e-Revolution

(C)  Phone to all

(D)  Smart phone to all

(E)  None of the above

Answer: (D)

11. Microfinance Institution ‘Janalakshmi Financial Services (JFS)’ has recently received final license from the Reserve Bank of India to set up a Small Finance Bank (SFB). The JFS is headquartered in-

(A)  Bengaluru, Karnataka

(B)  Patna, Bihar

(C)  Raipur, Chhattisgarh

(D)  Shimla, Himachal Pradesh

(E)  Delhi

Answer: (A)

12. India’s rank in the 2017 Global Connectivity Index stands at-

(A)  40th

(B)  41st

(C)  42nd

(D)  43rd

(E)  45th

Answer: (D)

13. BHEL’ is a public sector undertaking which manufactures a variety of heavy equipment. The alphabet ‘E’ in the abhreviation ‘BHEL’ stands for-

(A)  Electricals

(B)  Electronics

(C)  Election

(D)  Education

(E)  Enemy

Answer: (A)

14. The ‘SARFAESI Act 2002’ allows banks and financial institutions to auction residential or commercial properties to recover loans. The alphabet ‘R’ in the abbreviation ‘SARFAESI’ stands for-

(A)  Recover

(B)  Recognize

(C)  Restriction

(D)  Retention

(E)  Reconstruction

Answer: (E)

15. World Blood Donor Day is celebrated every year by the people in many countries around the world on which date?

(A)  14 September

(B)  8 March

(C)  15 April

(D)  14 June

(E)  15 July

Answer: (D)

16. ‘Janalakshmi Small Finance Bank’ is headquartered in-

(A)  Mumbai

(B)  Hyderabad

(C)  Delhi

(D)  Bengaluru

(E)  Kolkata

Answer: (D)

17. The headquarters of World Intellectual Property Organization is in which of the following country?

(A)  India

(B)  USA

(C)  Switzerland

(D)  England

(E)  Istanbul, Turkey

Answer: (C)

18. ‘Pocket’, is a digital wallet launched by-

(A)  HDFC Bank

(B)  ICICI Bank

(C)  AXIS Bank

(D)  SBI

(E)  PNB

Answer: (B)

19. The city ‘Dibrugarh’ is situated on the banks of the river-

(A)  Indus

(B)  Brahmpatura

(C)  Teeshta

(D)  Meghana

(E)  Ganga

Answer: (B)

20. According to Reserve Bank of India guidelines, the deadline for Indian banks to meet capital requirements under the Basel III norms is-

(A)  31st March, 2019

(B)  31st March, 2018

(C)  31st March, 2020

(D)  31st March, 2021

(E)  31st March, 2022

Answer: (A)

21. According to the RBI guidelines, the initial promoter stake in case of NOFHC should not be less than 40 per cent, locked for a period of-

(A)  2 years

(B)  3 years

(C)  4 years

(D)  5 years

(E)  6 years

Answer: (D)

22. The Union Government has constituted a committee to form a strategy to expedite the process of transforming India into a cashless economy. The name of the committee is-

(A)  Tarapore Committee

(B)  Shah Committee

(C)  Amitabh Kant Committee

(D)  Bimal Julka Committee

(E)  Bibek Debroy Committee

Answer: (C)

23. Manas Wildlife Sanctuary is a national park situated in the Indian State of-

(A)  Arunachal Pradesh

(B)  Assam

(C)  Madhya Pradesh

(D)  Uttar Pradesh

(E)  Uttarakhand

Answer: (B)

24. The ‘Digital Wallet Pockets’ is a complete payment solution launched by-

(A)  State Bank of India

(B)  ICICI Bank

(C)  IDFC Bank

(D)  AXIS Bank

(E)  HDFC Bank

Answer: (B)

25. The new base year adopted by CSO for releasing GDP data is-

(A)  2009-10

(B)  2010-11

(C)  2011-12

(D)  2014-15

(E)  2015-16

Answer: (C)

26. ‘Kathakali’ originated in the region around what is now covered by the Indian State of-

(A)  Gujarat

(B)  Odisha

(C)  Kerala

(D)  Tamil Nadu

(E)  Andhra Pradesh

Answer: (C)

27. The India’s longest cable-stayed bridge was inaugurated by PM Modi in which of the following states?

(A)  Madhya Pradesh

(B)  Gujarat

(C)  Punjab

(D)  Haryana

(E)  Maharashtra

Answer: (B)

28. Central Bank of India is a public sector bank headquartered in-

(A)  Mumbai, Maharashtra

(B)  Kolkata, West Bengal

(C)  Hyderabad, Telangana

(D)  New Delhi, Delhi

(E)  Bengaluru, Karnataka

Answer: (A)

29. Which of the following statements regarding Non-banking Finance Companies (NBFCs) is ‘not’ true?

(A)  The NBFCs are allowed to accept/renew public deposits for a minimum period of 12 months and a maximum period of 60 months.

(B)  The NBFCs cannot offer gifts or any other additional benefit the depositors.

(C)  The NBFCs can accept demand deposits repayable through cheques.

(D)  The deposits with NBFCs are not insured.

(E)  None of the above

Answer: (C)

30. Which of the following teams has recently won the UEFA Champions League?

(A)  Juventus

(B)  FC Barcelona

(C)  Real Madrid

(D)  Atletico Madrid

(E)  Monaco

Answer: (C)

31. World’s largest Uranium reserves are found in-

(A)  Canada

(B)  Chile

(C)  Zaire

(D)  Nigeria

(E)  Sweden

Answer: (A)

32. BSBDA is a Zero Balance Savings Account that takes care of your simple banking needs with Free ATM card, monthly statement, and cheque book. The alphabet ‘S’ in the abbreviation ‘BSBDA’ stands for-

(A)  Small

(B)  Sum

(C)  Savings

(D)  Short

(E)  System

Answer: (C)

33. Who is the incumbent governor of RBI?

(A)  Raghuram Rajan

(B)  S. S. Mundra

(C)  Viral Acharya

(D)  N. S. Vishwanathan

(E)  Dr. Urjit Patel

Answer: (E)

34. Which of the following Indian athletes had won silver medal in Rio Olympics, 2016?

(A)  Sakshi Malik

(B)  Saina Nehwal

(C)  Sourabh Varma

(D)  Srikanth Kidambi

(E)  P. V. Sindhu

Answer: (E)

35. UPI enables bank account holders (of banks participating in UPI) to send and receive money using a VPA without entering additional bank information. What is the full form of VPA?

(A)  Virtual Payment Address

(B)  Virtual Portal Address

(C)  Very Prominent Address

(D)  Very Payment Address

(E)  Very Portal Address

Answer: (A)

36. Which of the following is the India’s first state to get World’s largest roof top solar power plant?

(A)  Kerala

(B)  Punjab

(C)  Sikkim

(D)  Karnataka

(E)  Bihar

Answer: (B)

37. Who is the President of ‘Republic of Turkey’?

(A)  Serzh Sargsyan

(B)  Recep Tayyip Erdogan

(C)  Hassan Rouhani

(D)  Fuad Masum

(E)  Ashraf Ghani

Answer: (B)

38. What is the current repo rate?

(A)  6.75%

(B)  6.25%

(C)  5.75%

(D)  6.50%

(E)  6.0%

Answer: (B)

39. Which of the following is honoured with Vyas Samman Award 2017 by the PM Narendra Modi?

(A)  Surinder Verma

(B)  Sunita Jain

(C)  Vishwanath Tripathi

(D)  Rishabh Jain

(E)  Amar Kanth

Answer: (A)

40. Planning Commission of India was set up in the year-

(A)  1949

(B)  1957

(C)  1950

(D)  1951

(E)  1952

Answer: (C)

41. The Reserve Bank of India has constituted which committee to study various facets of household Finance in India?

(A)  Abhijit Sen Committee

(B)  C. B. Bhave Committee

(C)  Vasudev Committee

(D)  Tarun Ramadorai Committee

(E)  Talwar Committee

Answer: (D)

42. Who had won the 2016 Wimbledon Open in Women Singles?

(A)  Angelique Kerber

(B)  Garbine Muguruza

(C)  Agnieszka Radwanska

(D)  Serena Williams

(E)  Simona Halep

Answer: (D)

43. Which of the following is the dimension of the new Rs 2000 note?

(A)  63 mm ↔ 150 mm

(B)  66 mm ↔ 166 mm

(C)  55 mm ↔ 88 mm

(D)  60 mm ↔ 160 mm

(E)  64 mm ↔ 175 mm

Answer: (B)

44. The Federation of Indian Chambers of Commerce and Industry (FICCI) is headquartered in which of the following states?

(A)  New Delhi, Delhi

(B)  Bengaluru, Karnataka

(C)  Mumbai, Maharashtra

(D)  Kolkata, West Bengal

(E)  Bhopal, Madhya Pradesh

Answer: (A)

45. Whenever a transaction is completed and the cardholder (or his or her credit card) is not physically present to hand to the seller, it is known as CNP. What is the full form of CNP?

(A)  Care Never Present

(B)  Card Net Payment

(C)  Card Not Present

(D)  Card Not Payment

(E)  Card Never Payment

Answer: (C)

46. Synoptic Information is a leading System Integrator. The chairman of the company is-

(A)  Issa M. Allsaa

(B)  Ibrahim A. Al Touq

(C)  Nemah Sabagh

(D)  Mohammed Al Mandil

(E)  Mohammed Ali Jinnah

Answer: (D)

47. A joint naval exercise named Malabar-2017 is a joint military exercise involving-

(A)  India and Nepal

(B)  Nepal and Bhutan

(C)  India, Japan and U.S.A.

(D)  India and Myanmar

(E)  India and China

Answer: (C)

48. ‘Jitu Rai’ a gold medalist in ISSF, is associated with which of the following sports?

(A)  Boxing

(B)  Wrestling

(C)  Cricket

(D)  Shooting

(E)  Football

Answer: (D)

49. We can recharge, pay bills, send money, shop, get offers and more using Pockets App. Which of the following banks launched this application?

(A)  HDFC Bank

(B)  ICICI Bank

(C)  AXIS Bank

(D)  State Bank of India

(E)  Canara Bank

Answer: (B)

50. After the merger of SBI with its associated banks, the assets of SBI had increased to which of the following ?

(A)  Rs 41 lakh crore

(B)  Rs 45 lakh crore

(C)  Rs 60 lakh crore

(D)  Rs 70 lakh crore

(E)  Rs 80 lakh crore

Answer: (A)

Bank of Baroda PO Examination Held on 27-5-2017 Quantitative Aptitude Question Paper With Answer Key

Bank of Baroda PO Examination Held on 27-5-2017
Bank of Baroda PO Examination Held on 27-5-2017 Quantitative Aptitude Question Paper With Answer Key

Bank of Baroda PO Examination Held on 27-5-2017

Quantitative Aptitude

Directions – (Q. 1 to 5) In the following questions two equations numbered I and II are given. You  have to both the equations and give answer if-

(A) x < y

(B) x > y

(C) x ≤ y

(D) x ≥ y

(E)  x = y, or the relationship cannot be established

1. (I) 2x2 – 15x + 28 =0

(II) y2 – 7y + 12 = 0

Answer: (E)

2. (I) 2x2 + 3x + 1 = 0

(II) 8y2 + 22y + 15 = 0

Answer: (B)

3. (I) x2 + 8x + 15 = 0

(II) 2y2 + 7y + 3 = 0

Answer: (C)

4. (I) 2x2 – 11x + 15 = 0

(II) 2y2 – 17y + 35 = 0

Answer: (A)

5. (I) 2x2 – 15x + 27 = 0

(II) y2 = 9

Answer: (D)

Directions – (Q. 6 to 10) Study the following information carefully to answer the given question.

   The total population of three villages A, B and C together is 80,000 and the ratio between their respective populations is 5 : 4 : 7.

Out of the total population of village A, the age of 16% of villagers is equal to 61 years or more, the age of 36% of villagers is equal to or more than 31 years but less than 61 years and the age of the remaining villagers is less than 31 years.

Out of the total population of village B, the age of one-fifth of villagers is equal to 61 years or more. Out of the remaining population, the age of 12/25th villagers is equal to or more than 31 years but less than 61 years and the age of the remaining villagers is less than 31 years.

In village C, the number of villagers whose age is equal to 61 years or more is 50% more than the difference between the number of villagers in village A whose age is less than 31 years and that in village B. The number of villagers whose age is equal to or more than 31 years but less than 61 years is 80% more than that in village A in the same age group. The age of the remaining villagers is less than 31 years.

6. In village C, out of the total population whose total age is equal to 61 years or more, one-third of the females and out of the total population whose age is equal to or more than 31 years but less than 61 years, one-fifth are females. What is the respective ratio between the number of males whose age is equal to 61 years or more and the number of males whose age is equal to or more than 31 years but less than 61 years?

(A)  27 : 85

(B)  23 : 81

(C)  22 : 81

(D)  23 : 83

(E)  25 : 64

Answer: (B)

7. In village A, the respective ratio between the total number of male villagers and the total number of female villagers is 23 : 27. If 9/16th of the total villagers whose age is equal to 61 years or more are females, what per cent of female population in village A is equal to 61 years or more?

(A) 

(B) 

(C)  

(D) 

(E) 

Answer: (D)

8. In village B, out of the total population, 36% are graduates. If the number of male graduates is 40% more than that of female graduates, females graduates are what per cent of the total population?

(A)  15

(B)  20

(C)  5

(D)  25

(E)  18

Answer: (A)

9. What is the average population across the three villages whose age is less than 31 years?

(A)  11500

(B)  11750

(C)  12500

(D)  12700

(E)  11200

Answer: (E)

10. By what per cent the population of village B whose age is equal to or greater than 31 years, is less than the population of village A in the same age group?

(A) 

(B) 

(C) 

(D) 

(E) 

Answer: (E)

Directions-(Q. 11to 14) What should come in place of the question mark (?) in the following number series?

  1. 6 4  4  8  48  ?

(A)  736

(B)  720

(C)  744

(D)  742

(E)  748

Answer: (A)

12. 65  33  17  9  5  ?

(A)  4

(B)  4.5

(C)  5

(D)  3

(E)  2.5

Answer: (D)

13. 22  13  24  11  26  ?

(A)  9

(B)  15

(C)  12

(D)  10

(E)  8

Answer: (A)

14. 185   149  119  94  73  ?

(A)  52

(B)  42

(C)  55

(D)  58

(E)  48

Answer: (C)

15. In an examination, the number of students who passed and the number of those who failed were in the ratio of 25 : 4 respectively. If five more had appeared and the number of failures was 2 less than earlier, the ratio of passed to failed would have been 22 : 3. What is the number of students who appeared for the examination ?

(A)  145

(B)  150

(C)  155

(D)  180

(E)  190

Answer: (A)

16. The ratio of A to B is 4 : 5 and that of B to C is 2 : 3. If A equals 800, then what is the value of C?

(A)  1000

(B)  1200

(C)  1500

(D)  2000

(E)  3000

Answer: (C)

Directions-(Q. 17 to 21) Study the following graph carefully and answer the questions given below-

17. Number of projects handled by company T increased by what per cent from 2003 to 2004?

(A) 

(B) 

(C) 

(D) 

(E)

Answer: (D)

18. Number of projects handled by companies R and S increase by 25% and 16% respectively from 2004 to 2005. What was the total number of projects handled by companies R and S together in 2005?

(A)  524

(B)  548

(C)  532

(D)  518

(E)  None of these

Answer: (C)

19. Number of project by company R in 2006 was 150% of the number of projects handled by it in 2003 and number of projects handled by company S in 2006 was 60% of the number of projects handled by it in 2003. What is the respective ratio between number of projects handled by company R and S in 2006?

(A)  9 : 5

(B)  12 : 7

(C)  8 : 3

(D)  18 : 7

(E)  18 : 5

Answer: (D)

20. Combining years 2003 and 2004, 58% of the total number of projects handled by company P was non-governmental and 35% of the total number of projects handled by company Q was non-governmental. What is the total number of non-governmental projects handled by companies P and Q in 2003 and 2004 together?

(A)  623

(B)  641

(C)  559

(D)  659

(E)  595

Answer: (C)

21. What is the average number of projects handled by P, Q and T in 2003?

(A)  290

(B)  320

(C)  305

(D)  310

(E)  315

Answer: (D)

22. A and B entered into a partner ship, investing Rs 16,000 and Rs 12,000 respectively. After 3 months, ‘A’ withdrew Rs 5000, while B’ invested Rs 5000 more. After 3 month more, C joins the business with a capital of Rs 21,000. After a year, they obtained a profit of Rs 26,400. By what amount does the profit of B exceed the share of C?

(A)  Rs 3600

(B)  Rs 3800

(C)  Rs 4600

(D)  Rs 4800

(E)  Rs 5000

Answer: (A)

23. On what sum will the difference between the simple and compound interest for 3 years at 5 per cent per annum amount to Rs 24.40?

(A)  3300

(B)  3587

(C)  320

(D)  2800

(E)  3000

Answer: (C)

24. If 6 years are subtracted from the present age of Randheer and the remainder is divided by 18, then the present age of his grandson Anup is obtained. If Anup is 2 years younger to Mahesh whose age is 5 years, then what is the age of Randheer?

(A)  96 years

(B)  84 years

(C)  48 years

(D)  60 years

(E)  66 years

Answer: (D)

25. If 2 kg of metal, of which 1/3 is zinc and the rest is copper, be mixed with 3 kg of metal, of which 1/4 zinc and the rest is copper, what is the ratio of zinc to copper in the mixture?

(A)  13 : 42

(B)  17 : 43

(C)  19 : 43

(D)  15 : 42

(E)  16 : 43

Answer: (B)

Directions – (Q. 26 to 29) Each of the questions given below consist of question and two statements numbered I and II given below it. You have to decide whether the data provided in the statements are sufficient to answer the question and give answer :

(A) If the data in statement I alone is sufficient to answer the question, while the data in statement II alone is not sufficient to answer the question.

(B) If the data in statement II alone is sufficient to answer the question, while the data in statement I alone is not sufficient to answer the question.

(C) If the data in statement I alone or in statement II alone is sufficient to answer the question.

(D) If the data in both the statements I and II is not sufficient to answer the question.

(E) If the data in both the statements I and II together is necessary to answer the question.

26. If ‘X’ litre was removed from jar containing a mixture of milk and water in the ratio of 4 : 1 respectively, what was value of X?

(I) After removing ‘X’ litre of mixture from the jar, 16 litre of water was added to the jar and the respective ratio between milk and water (in the jar) became 2 : 1.

(II) ‘X’ litre of the mixture constituted 20% of the original quantity of mixture in the jar.

Answer: (E)

27. If the average of 3 positive integers A, B and C is 92, what is the value of A?

(I) C – B = 36

(II) If X = A – B, then (C – X) – (X – B) = 132

Answer: (E)

28. If Ram’s mother is 5 years younger than his father, what is the respective ratio between Ram’s age 4 years hence and his father’s age 12 years ago ?

(I) Seven years ago, Ram’s father was 55 years old.

(II) The respective ratio between the Ram’s age 3 years hence and his mother’s age 3 years hence will be 2 : 5.

Answer: (E)

29. A, B and C start a business with initial investments in the respective ratio of 3 : 4 : 2. Four months after the start of the business, B halves his investment. What was C’s investment?

(I) The difference between B’s and C’s share of the annual profit was Rs 600.

(II) The ratio between the annual investments by A, B and C was 9 : 8 : 6 respectively.

Answer: (A)

30. Aniket deposited two parts of a sum of Rs 25000 in different banks at the rates of 15% per annum and 18% per annum respectively. In one year he got Rs 4050 as the total interest. What was the amount deposited at the rate of 18% per annum?

(A)  Rs 10000

(B)  Rs 18000

(C)  Rs 15000

(D)  Rs 12000

(E)  Rs 16000

Answer: (A)

31. One bag contains 4 white balls and 2 black balls. Another bag contains 3 white balls and 5 black balls. If one ball is drawn from each bag, what is the probability that one ball is white and another is black?

(A)  6/24

(B)  5/24

(C)  7/24

(D)  13/24

(E)  14/24

Answer: (D)

32. A and B together can do a work in 8 days, B and C together in 6 days, while C and A together in 10 days. If they all work together, in how many days will they complete the work?

(A) 

(B) 

(C) 

(D) 

(E) 

Answer: (C)

Directions – (Q. 33 to 36) Refer to the pie chart and answer the given question.

33. What is the average quantity of rice distributed to Retailers B, C and D? (in kg)

(A)  136

(B)  142

(C)  140

(D)  138

(E)  134

Answer: (A)

34. What is the central angle corresponding to the quantity of rice distributed to retailer C?

(A)  68.4°

(B)  67.6°

(C)  69.4°

(D)  65.5°

(E)  None of these

Answer: (A)

35. What is the difference between the total quantity of rice distributed to retailers A and B together and that distributed to Retailers D and E together?

(A)  32

(B)  42

(C)  36

(D)  40

(E)  38

Answer: (D)

36. In April, 2017, if the total quantity of rice in the granary increased by 5% over the previous month and the additional quantity of rice was distributed to retailers B and E only, what was the total quantity of rice distributed to Retailers B and E together? (in kg)

(A)  398

(B)  406

(C)  476

(D)  356

(E)  376

Answer: (E)

37. A boat travels from A to B upstream and then from B to C downstream taking the same time. The respective ratio between the distance from A to B and the distance from B to C is 5 : 7. If the boat takes 2 hours 30 minutes to travel a distance of 35 km downstream, what is the speed of the stream? (in km/hr.)

(A)  2 km/hr.

(B)  3 km/hr.

(C)  12 km/hr.

(D)  10 km/hr.

(E)  14 km/hr.

Answer: (A)

38. A boat takes 19 hr. for travelling downstream from point A to point B and coming back to a point C midway between A and B. If the velocity of the stream is 4 km/hr. and the speed of the boat in still water is 14 km/hr., what is the distance between A and B?

(A)  200 km

(B)  180 km

(C)  160 km

(D)  220 km

(E)  210 km

Answer: (B)

39. A person purchases 100 pens at a discount of 10%. The new amount of money spent by the person to purchase the pens is Rs 600. The selling expenses incurred by the person are 15% on the net cost price. What should be the selling price for 100 pens in order to earn a profit of 25%?

(A)  802.50

(B)  811.25

(C)  862.50

(D)  875

(E)  825

Answer: (D)

Directions – (Q. 40 to 44) What approximate value should come in place of question-mark (?) in the following question ? (You are not expected to calculate the exact value).

40. 9.0013 = (? – 3.3) * 89.897 ÷ ∜624.99

(A)  53

(B)  16

(C)  28

(D)  44

(E)  12

Answer: (D)

41. 

(A)  34

(B)  49

(C)  32

(D)  38

(E)  42

Answer: (C)

42. 

(A)  125

(B)  120

(C)  100

(D)  81

(E)  64

Answer: (C)

43. 

(A)  4

(B)  5

(C)  9

(D)  10

(E)  16

Answer: (E)

44. 

(A)  125

(B)  110

(C)  250

(D)  50

(E)  75

Answer: (A)

45. The ratio between the length and the breath of a rectangular park is 3 : 2. If a man cycling along the boundary of the park at the speed of 12 km/hr completes one round in 8 minutes, then what is the area of the park?

(A)  154000

(B)  153600

(C)  307400

(D)  307200

(E)  412500

Answer: (B)

Directions – (Q. 46 to 50) Study the table and answer the given question. Data regarding number of people (both literate and illiterate who attended a workshop.

46. Total number of people (literates + Illiterates) who attended the workshop on Sunday was what per cent more than those who attended on Thursday?

(A)  7

(B)  15

(C)  None

(D)  10

(E)  5

Answer: (D)

47. On Thursday, if 192 illiterate males attended the workshop, what was the number of literate females who attended the work-shop on the day?

(A)  292

(B)  314

(C)  296

(D)  308

(E)  298

Answer: (A)

48. On Friday, if the number of illiterates (Male + Females) increased by 40% and those of literates (Males + Females) reduced by 20%, as compared to Monday, what was the difference between the number of literates and illiterates who attended the workshop on Friday?

(A)  18

(B)  24

(C)  22

(D)  48

(E)  14

Answer: (E)

49. What is the average number of illiterates (Male + Females) who attended the workshop on Sunday, Tuesday and Wednesday?

(A)  370

(B)  340

(C)  350

(D)  360

(E)  380

Answer: (A)

50. What is the respective ratio between the total number of males (Literates + Illiterates) who attended the workshop on Monday and Tuesday together and that of females (Literates and Illiterates) who attend the workshop on the same days together?

(A)  5 : 6

(B)  7 : 9

(C)  4 : 7

(D)  7 : 5

(E)  5 : 7

Answer: (B)

Bank of Baroda PO (Scale-I) Examination Held on 18-4-2015 Quantitative Aptitude Question Paper With Answer Key

Bank of Baroda PO (Scale-I) Examination Held on 18-4-2015 Quantitative Aptitude
Bank of Baroda PO (Scale-I) Examination Held on 18-4-2015 Quantitative Aptitude Question Paper With Answer Key

Bank of Baroda PO (Scale-I) Examination Held on 18-4-2015

Quantitative Aptitude

Directions – (Q. 1 to 5) In this question two equations numbered I and II are given. You have to solve both the equations and mark the appropriate option.

1. I. 2x2 – x – 10 = 0

II. 2y2 – y – 21 = 0

(A)  x < y

(B)  x > y

(C)  x ≤ y

(D)  x ≥ y

(E)  Relationship between x and y cannot be established

Answer: (E)

2. I. 2x2 + 11x + 15 = 0

II. 4y2 + 22y + 24 = 0

(A)  x < y

(B)  x > y

(C)  x ≤ y

(D)  x ≥ y

(E)  Relationship between x and y cannot be established

Answer: (E)

3. I. 2x2 + 9x + 9 = 0

II. 2y2 + 17y + 36 = 0

(A)  x < y

(B)  x > y

(C)  x ≤ y

(D)  x ≥ y

(E)  Relationship between x and y cannot be established

Answer: (B)

4. I. 3x2 – 22x + 40 = 0

II. 2y2 – 19y + 44 = 0

(A)  x < y

(B)  x > y

(C)  x ≤ y

(D)  x ≥ y

(E)  Relationship between x and y cannot be established

Answer: (C)

5. I. 3x2 – 16x + 21 = 0

II. 3y2 – 28y + 65 = 0

(A)  x < y

(B)  x > y

(C)  x ≤ y

(D)  x ≥ y

(E)  Relationship between x and y cannot be established

Answer: ()

6. A merchant buys two items for Rs 7,500. One item he sells at a profit of 16% and the other item at 14% loss. In the deal the merchant makes neither any profit nor any loss. What is the difference between selling price of both the items? (in Rupees)

(A)  625

(B)  610

(C)  620

(D)  630

(E)  615

Answer: (C)

7. There was 120 litres of pure milk in a vessel. Some quantity of milk was taken out and replaced with 23 litres of water in such a way that the resultant ratio between the quantities of milk and water in the mixture was 4 : 1 respectively. Again 23 litres of the mixture was taken out and replaced with 27 litres of water. What is the respective ratio of milk and water in the resultant mixture?

(A)  58 : 37

(B)  116 : 69

(C)  69 : 43

(D)  101 : 37

(E)  None of these

Answer: (E)

Directions – (Q. 8-12) Refer to the pie-charts and answer the given question-

8. If the respective ratio between total number of male professors and total number of female professors is 9 : 16 ad the respective ratio  between total number of male assistant professors and total number of female assistant professors is 9 : 11, what is the total number of female professors and assistant professors teaching the given six subjects in University ‘X’?

(A)  63

(B)  90

(C)  77

(D)  73

(E)  87

Answer: (E)

9. What is the central angle corresponding to the total number of professors and assistant professors teaching Hindi ?

(A)  46.8°

(B)  50.4°

(C)  43.2°

(D)  39.6°

(E)  38.4°

Answer: (E)

10. What per cent professors are teaching Psychology and Sociology together out of the total number of Professors and assistant Professors teaching these two subjects together?

(A) 

(B) 

(C) 

(D) 

(E) 

Answer: (A)

11. The total number of assistant professors teaching Economics and English together are what per cent more than the total number of professors teaching these two subjects together?

(A) 

(B) 

(C) 

(D) 

(E) 

Answer: (C)

12. What is the average number of professors teaching Computer Science, Psychology, English and Sociology ?

(A)  8.5

(B)  8

(C)  10

(D)  10.5

(E)  17

Answer: (A)

13. 15 years ago the average age of a family of four members was 40 years. Two children were born in that span of 15 years. The present average age of the family remained unchanged. Among the two children who were born in between the 15 years. If the older child at present is 8 years more than the younger one, what is the respective ratio between the present age of the older child and the present age of the younger child ?

(A)  9 : 4

(B)  7 : 3

(C)  7 : 6

(D)  7 : 4

(E)  9 : 5

Answer: (B)

Directions – (Q. 14-18) The question consist of a question and two statements I and II given below it. You have to decide whether the data provided in the statements are sufficient to answer the question. Read both the statements and choose the appropriate option.

14. What is the definitely the value of ‘x’?

I. The value of  is equal to 0.

II. The value of 3x – 2 . 92x – 3 is equal to 95x – 19

(A)  The data either in statement I alone or in statement II alone are sufficient to answer the question

(B)  The data in both the statement I and II together are not sufficient to answer the question

(C)  The data in both the statements I and II together are necessary to answer the question

(D)  The data in statement I alone are sufficient to answer the question while the data in statement II alone are not sufficient to answer the question

(E)  The data in statement II alone are sufficient to answer the question while the data in statement I alone are not sufficient to answer the question

Answer: (E)

15. An item was sold after giving a certain discount on the marked price. What was the percentage of discount given ?

I. The profit earned by after giving the discount is 44%. Had the percentage of discount been doubled the profit earned would have been 28%.

II. The cost price of the item is Rs 400.

(A)  The data either in statement I alone or in statement II alone are sufficient to answer the question

(B)  The data in both the statement I and II together are not sufficient to answer the question

(C)  The data in both the statements I and II together are necessary to answer the question

(D)  The data in statement I alone are sufficient to answer the question while the data in statement II alone are not sufficient to answer the question

(E)  The data in statement II alone are sufficient to answer the question while the data in statement I alone are not sufficient to answer the question

Answer: (D)

16. What is the respective ratio of initial investments of A and B?

I. A started the business by investing a certain amount and he invested for the whole year. B joined A after 4 months from the start of the business and invested for the rest of the year. The profit earned by A and B are in the respective ratio 7 : 8.

II. The initial investment of B was Rs 10,800.

(A)  The data either in statement I alone or in statement II alone are sufficient to answer the question

(B)  The data in both the statement I and II together are not sufficient to answer the question

(C)  The data in both the statements I and II together are necessary to answer the question

(D)  The data in statement I alone are sufficient to answer the question while the data in statement II alone are not sufficient to answer the question

(E)  The data in statement II alone are sufficient to answer the question while the data in statement I alone are not sufficient to answer the question

Answer: (D)

17. How much will the boat take to cover a distance of 63 km upstream?

I. The difference between the time taken by the boat to travel from A to B (upstream) and time taken by it to travel from B to A ( downstream) is 2 hours.

II. The distance between A and B is 45 km and speed of the boat in still water is 12 kmph.

(A)  The data either in statement I alone or in statement II alone are sufficient to answer the question

(B)  The data in both the statement I and II together are not sufficient to answer the question

(C)  The data in both the statements I and II together are necessary to answer the question

(D)  The data in statement I alone are sufficient to answer the question while the data in statement II alone are not sufficient to answer the question

(E)  The data in statement II alone are sufficient to answer the question while the data in statement I alone are not sufficient to answer the question

Answer: (C)

18. There are two cylindrical rollers bigger and smaller. How many rotations will the bigger roller take to flatten a stretch of land (X) ?

I. The respective ratio of the radii of the bigger and the smaller roller is 7 : 3. Both the rollers are of the same length.

II. The smaller takes 63 rotations to flatten the stretch of land (X).

(A)  The data either in statement I alone or in statement II alone are sufficient to answer the question

(B)  The data in both the statement I and II together are not sufficient to answer the question

(C)  The data in both the statements I and II together are necessary to answer the question

(D)  The data in statement I alone are sufficient to answer the question while the data in statement II alone are not sufficient to answer the question

(E)  The data in statement II alone are sufficient to answer the question while the data in statement I alone are not sufficient to answer the question

Answer: (C)

19. A man takes 17/9 times as long to row a distance upstream as to row the same distance downstream. What is the speed of the boat in still water if it takes 3 hours to travel 38.4 km downstream ? (in km/h)

(A)  11

(B)  10

(C)  12.4

(D)  12

(E)  14

Answer: (B)

20. In a company ‘XYZ’, the respective ratio between the total number of under-graduate employees and the total number of graduate employees is 13 : 23. The Company has only two branches, one in Mumbai and one in Delhi. If the total number of under-graduate employees in Mumbai branch is 351, which is 30% of the total undergraduate employees in the company, what is the total number of graduate employees in the company?

(A)  2185

(B)  1955

(C)  2070

(D)  2100

(E)  1985

Answer: (C)

Directions- (Q. 21-25) Which of the following will come in  place of the question-mark (?) in the given question ?

21. (30.06% of 560.14 + 53.02% of 1100)/8 = ?

(A)  78

(B)  94

(C)  99

(D)  81

(E)  85

Answer: (B)

22. ? × 5 × 4.92 – 13.13 × 4.02 × 4 = 117

(A)  18

(B)  13

(C)  7

(D)  21

(E)  9

Answer: (B)

23. (9117.88 – 8021.85 + 93.92) × 12 = 1500 × ?

(A)  12

(B)  8

(C)  15

(D)  22

(E)  4

Answer: (C)

24. 

(A)  729

(B)  1849

(C)  2209

(D)  1369

(E)  1089

Answer: (E)

25. 5 × 5.04 + 237 – 302.11 = ?

(A)  5125

(B)  5000

(C)  5035

(D)  5005

(E)  5085

Answer: (C)

Directions – (Q. 26-30) Study the table to answer the given questions-

26. What is the difference between the total number of cars manufactured in countries A and B together and the total number of cars manufactures in country C and D together?

(A)  10,000

(B)  12,000

(C)  15,000

(D)  6,000

(E)  10,500

Answer: (A)

27. Total number of cars sold in countries A and B together for what per cent of the total cars manufactured by in all the countries together in that particular quarter?

(A)  30

(B)  35

(C)  40

(D)  32

(E)  38

Answer: (B)

28. What is the total number of cars which remained unsold in the given quarter in the countries C and E together?

(A)  2400

(B)  3400

(C)  3650

(D)  3800

(E)  3600

Answer: (E)

29. What is the respective ratio between sales (in Rs) of the total number of cars sold in country D and the sales (in Rs) of the total number of cars sold in country E ?

(A)  55 : 47

(B)  52 : 47

(C)  53 : 44

(D)  55 : 42

(E)  52 : 45

Answer: (D)

30. If the cost for manufacturing a car in country C is Rs 4,50,000, what is the per cent profit earned by the company on which car sold in the same company?

(A) 

(B) 

(C) 

(D) 

(E) 

Answer: (E)

Directions – (Q. 31-35) Refer to the graph and answer the given question-

31. Number of elephants in reserve A increased by what per cent from 2001 to 2005 ?

(A)  56.5

(B)  62.5

(C)  68.5

(D)  54.5

(E)  58.5

Answer: (B)

32. What is the average number of elephants in reserve B during 2001, 2003, 2005 and 2006?

(A)  46

(B)  56

(C)  51

(D)  53

(E)  49

Answer: (C)

33. Out of the total number of elephants in reserves A and B together in 2002 only 40% were African elephants and out of the total number of elephants in reserves A and B together in 2004 only 65% were African elephants. What is the difference between total number of African elephants in reserves A and B together in 2002 and total number of African elephants in reserves A and B together in 2004 ?

(A)  37

(B)  47

(C)  43

(D)  41

(E)  55

Answer: (A)

34. What is the respective ratio between total number of elephants in reserves A in 2003 and 2006 together and total number of elephants in reserve B in 2005 and 2006 together?

(A)  17 : 18

(B)  23 : 24

(C)  11 : 14

(D)  9 : 14

(E)  11 : 12

Answer: (E)

35. Number of elephants in reserve A decreased by 25% from 2006 to 2007 and number of elephants in reserve B increased by 15% from 2006 to 2007. What was the total number of elephants in reserves A and B together in 2007?

(A)  172

(B)  164

(C)  158

(D)  174

(E)  166

Answer: (B)

36. There are two garbage disposal rectangular tanks, A and B with lengths 12 m and 15 m respectively in a square field. If the total area of the square field excluding the rectangular tanks is 360 sq. m and the breadth both the rectangular tanks is 1/3 of the side of the square field, what is the perimeter of the square field ? (in m)

(A)  92

(B)  84

(C)  96

(D)  78

(E)  72

Answer: (C)

37. Raman took a loan of Rs 15,000 from Laxman. He was agreed that for the first three years rate of interest charged would be at 8% Simple Interest per annum and at 10% Compound Interest (compounded annually) from the fourth year onwards. Ram did not pay anything until the end of the fifth year. How much would he repay if he clears the entire amount, only at the end of fifth year ? (in Rs)

(A)  Rs 22,506

(B)  Rs 22,105

(C)  Rs 22,900

(D)  Rs 22,500

(E)  Rs 22,450

Answer: (A)

Directions – (Q. 38-42) Study the information carefully  to answer the question :

Sidhartha has decidied to start a new company ‘Sidhartha Travels’. He wants to buy some furniture-table, chair, air-conditioners. He also wants to buy a few desktops which will include monitor, CPU, keyboard and mouse. The cost of each table is 13.5 times the cost of a mouse and the cost of a chair is 3/5th of the most of a table. The cost of an air-conditioner is 5 times the cost of a chair.

The cost of a monitor is 20% more than the cost of a chair, a CPU costs Rs 1,500 more than a table and the keyboard costs 4 times a mouse. The cost of a mouse is Rs 1,000.

38. What will be the total cost of 1 table and 2 chairs together?

(A)  23,200

(B)  28,800

(C)  29,700

(D)  24,700

(E)  21,300

Answer: (C)

39. What is the respective ratio between the cost of a CPU and the total cost of a mouse and a keyboard together?

(A)  3 : 2

(B)  4 : 1

(C)  3 : 1

(D)  5 : 1

(E)  5 : 3

Answer: (C)

40. If the cost of a keyboard and mouse increases by 20% and 15% respectively, what will be the total cost of a desktop (given that the cost of CPU and Monitor the same) ?

(A)  Rs 36,930

(B)  Rs 32,640

(C)  Rs 34,753

(D)  Rs 30,670

(E)  Rs 35,425

Answer: (D)

41. Sidharatha bought 1 table, 1 chair and 1 desktop for himself. What was the total cost incurred by him ?

(A)  Rs 53,400

(B)  Rs 51,460

(C)  Rs 50,640

(D)  Rs 55,440

(E)  Rs 51,320

Answer: (E)

42. It was initially decided that 4 air-conditioners will be installed but later only 3 air-conditioners and a fan were installed. If the cost of a fan is 1/10th of t he cost of air-conditioner, what was the total cost incurred?

(A)  Rs 1,29,400

(B)  Rs 1,24,500

(C)  Rs 1,25,550

(D)  Rs 1,24,350

(E)  Rs 1,25,600

Answer: (C)

43. Prem and Shyam decide to go on a trip to point Yon a particular from Point X. Prem leaves for Point Y at 11 : 00 am, at speed of 72 km/hour. Shyam leaves for point Y at 11 “ 30 same day as Prem left. At what speed should Shyam travel to catch up with Prem in 4 hours ? (in km/hour)

(A)  85

(B)  81

(C)  80

(D)  82

(E)  86

Answer: (B)

Directions- (Q. 44-48) What will come in place of question-mark (?) in the given numbers series?

44. 2, 12.9, 9.5, 14.6, 7.8, ?

(A)  17.9

(B)  16.3

(C)  16.7

(D)  16.2

(E)  16.9

Answer: (B)

45. 8, 5, 9, 22.5, 61, ?

(A)  184

(B)  181.5

(C)  192.5

(D)  177.5

(E)  172.5

Answer: (D)

46. 7, 9, 24, 84, ?, 1810

(A)  336

(B)  356

(C)  348

(D)  340

(E)  352

Answer: (B)

47. 11, 27, 48, 84, 145, ?

(A)  241

(B)  239

(C)  263

(D)  257

(E)  229

Answer: (A)

48. 16, 15, 20, 43, 102, ?

(A)  221

(B)  237

(C)  213

(D)  251

(E)  249

Answer: (A)

49. ‘A’ began a small business with a certain amount of money. After four months from the start of the business, ‘B’ joins the business with an amount which is Rs 6,000 less than ‘A’s initial investment. ‘C’ joins the business after seven months from the start of business with an amount which is Rs 2,000 less than A’s initial investment. At the end of the year total investment reported was Rs 1,42,000, what will be A’s share in the profit, If B received Rs 8,000 as profit share ? (in Rs)

(A)  Rs 48,000

(B)  Rs 46,500

(C)  Rs 46,000

(D)  Rs 42,000

(E)  Rs 48,500

Answer: (A)

50. If 6 men and 8 boys can do a piece of work in 10 days while 26 men and 48 boys can do the same in 2 days, How much time taken by 15 men and 20 boys in doing the same type of work?

(A)  4 days

(B)  5 days

(C)  6 days

(D)  7 days

(E)  8 days

Answer: (A)

Bank of Baroda Probationary Officers Examination-2015 Question Paper With Answer Key

Bank of Baroda Probationary Officers Examination-2015
Bank of Baroda Probationary Officers Examination-2015 Question Paper With Answer Key

Bank of Baroda Probationary Officers Examination-2015

Reasoning

 

Directions (Qs. 1 to5): These questions consist of a question and two statements numbered I and II given below it. You have to decide whether the data given in the statements are sufficient to answer the question. Read both the statements and choose the most appropriate option.

1. Among five friends, A, B, C, D and E each studying in a different standard viz. 1st, 4th, 7th 8th, and 10th, in which standard does C study? (‘Junior’ in the statements implies a lower standard and ‘senior’ higher standard)

(I) B studies in a standard which is an odd number. C is B’s senior but does not study in 10th standard.

(II) Only three people are senior to A. B is senior to A but junior to C.

(a)  The data in statement I alone are sufficient to answer the question, while the data in statement II alone are not sufficient to answer the question.

(b)  The data in statement II alone are sufficient to answer the question, while the data in statement I alone are not sufficient to answer the question.

(c)  The data either in statement I alone or in statement II alone are sufficient to answer the question.

(d)  The data in both statements I and II together are not sufficient to answer the question.

(e)  The data in both the statements I and II together are necessary to answer the question.

Answer: (e)

2. Four fiends viz. P, Q, R and S are seated around a circular table. Some are facing the centre while some face outside (i.e. opposite to the centre). Which direction is R facing (centre or outside)?

(I) R sits second to left of S. R sits to immediate right of Q.

(II) Q sits to immediate right of R. P is an immediate neighbour of both R and S.

(a)  The data in statement I alone are sufficient to answer the question, while the data in statement II alone are not sufficient to answer the question.

(b)  The data in statement II alone are sufficient to answer the question, while the data in statement I alone are not sufficient to answer the question.

(c)  The data either in statement I alone or in statement II alone are sufficient to answer the question.

(d)  The data in both statements I and II together are not sufficient to answer the question.

(e)  The data in both the statements I and II together are necessary to answer the question.

Answer: (d)

3. Point M is in which direction with respect to point A?

(I) A person starts walking from point A, walks 5 m towards North and then takes a right turn and walks 8m. He then takes left turn and walks 8m. he then takes a left turn and walks for 3 m before stopping at point Z. Point M is 12 m away from Point Z.

(II) A person starts from Point A and walks 9 m towards the West. he then takes a right turn and walks 9 m towards the West. He then takes a right turn and walks 5 m. he then takes a final right turn and stops at point R after walking for 5 m. Point M is towards the North of point R.

(a)  The data in statement I alone are sufficient to answer the question, while the data in statement II alone are not sufficient to answer the question.

(b)  The data in statement II alone are sufficient to answer the question, while the data in statement I alone are not sufficient to answer the question.

(c)  The data either in statement I alone or in statement II alone are sufficient to answer the question.

(d)  The data in both statements I and II together are not sufficient to answer the question.

(e)  The data in both the statements I and II together are necessary to answer the question.

Answer: (a)

4. Is Q the father of A?

(I) M is the mother of A and B. C is the only sister of A. R is married to B. R is the daughter-in-law of Q.

(II) Q has only one daughter C. C is the sister of A and B. B is the son of M. R is the daughter-in-law of M.

(a)  The data in statement I alone are sufficient to answer the question, while the data in statement II alone are not sufficient to answer the question.

(b)  The data in statement II alone are sufficient to answer the question, while the data in statement I alone are not sufficient to answer the question.

(c)  The data either in statement I alone or in statement II alone are sufficient to answer the question.

(d)  The data in both statements I and II together are not sufficient to answer the question.

(e)  The data in both the statements I and II together are necessary to answer the question.

Answer: (c)

5. Six people are sitting in two parallel rows containing three people each, in such a way that there is an equal distance between adjacent persons. In row-1, A, B and C are seated and all of them are facing South. In row-2, P, Q and R are seated and all of them are facing North. (Therefore, in the given seating arrangement each member seated in a row faces another member of the other row.) Who amongst A, B and C faces R?

(I)  A faces the one who sits to immediate right of R.. A is not an immediate neighbour of C.

(II) Only one person sits between Q and P. The one who faces B sits to immediate left of Q.

(a)  The data in statement I alone are sufficient to answer the question, while the data in statement II alone are not sufficient to answer the question.

(b)  The data in statement II alone are sufficient to answer the question, while the data in statement I alone are not sufficient to answer the question.

(c)  The data either in statement I alone or in statement II alone are sufficient to answer the question.

(d)  The data in both statements I and II together are not sufficient to answer the question.

(e)  The data in both the statements I and II together are necessary to answer the question.

Answer: (b)

6. If all he vowels in the word ‘SAINTLY’ are rearranged in the alphabetical order from left of right, followed by the consonants in the alphabetical order from left to right, the position of which of the following five alphabets given below will remain unchanged?

(a)  S

(b)  T

(c)  L

(d)  A

(e)  Y

Answer: (e)

Directions (Qs. 7 to 9): Read the information carefully and answer the given questions.

P @ Q means P is the father of Q.

P + Q means P is the husband of Q.

P $ Q means P is the brother of Q.

P % Q means P is the mother of Q.

P & Q means P is the sister of Q.

7. How is B related to E in this expression: ‘A @ B % C & D + E’?

(a)  Grandmother

(b)  Granddaughter

(c)  Mother-in-law

(d)  Aunt

(e)  Daughter-in-law

Answer: (c)

8. What should come in place of the question mark to establish that C is the aunt of E in the expression?

A % B + C & D ? E

(a)  $

(b)  &

(c)  +

(d)  Either + or &

(e)  Either @ or %

Answer: (e)

9. Which among the following options is true if the expression ‘A + B & C @ D % E $ F’ is definitely true?

(a)  E is the sister-in-law of A.

(b)  F is the daughter-in-law of C.

(c)  B is the aunt of D.

(d)  A is the uncle of E.

(e)  C is the uncle of A.

Answer: (c)

Directions (Qs. 10 to 15) : In these questions two/three statements followed by two conclusions numbered I and II have been given. You have to take the given statements to be true even if they seem to be at variance from commonly known facts and then decide which of the given conclusions logically follows from the given statements disregarding commonly known facts.

10. Statements:

All drivers are swimmers. Some swimmers are athletes. No athlete is a banker.

Conclusion I: All swimmer being bankers is a possibility.

Conclusion II: No driver is a banker.

(a)  Either conclusion I or II is true

(b)  Both conclusions I and II are true.

(c)  Only conclusion II is true.

(d)  Neither conclusion I nor II is true.

(e)  Only conclusion I is true.

Answer: (d)

11. Statements :

All frogs are amphibians. Some turtles are amphibians. All turtles are reptiles.

Conclusion I : At least some amphibians are reptiles.

Conclusion II : No frog is a turtle.

(a)  Either conclusion I or II is true

(b)  Both conclusions I and II are true.

(c)  Only conclusion II is true.

(d)  Neither conclusion I nor II is true.

(e)  Only conclusion I is true.

Answer: (e)

12. Statements:

All kings are warriors. Some dukes are kings.

Conclusion I : All kings are dukes.

Conclusion II : At least some dukes are warriors.

(a)  Either conclusion I or II is true

(b)  Both conclusions I and II are true.

(c)  Only conclusion II is true.

(d)  Neither conclusion I nor II is true.

(e)  Only conclusion I is true.

Answer: (c)

13. Statements:

Some plants are trees. All trees are weeds. All weeds are shrubs.

Conclusion I: All  plants are weeds.

Conclusion II: Some plants are weeds.

(a)  Either conclusion I or II is true

(b)  Both conclusions I and II are true.

(c)  Only conclusion II is true.

(d)  Neither conclusion I nor II is true.

(e)  Only conclusion I is true.

Answer: (c)

14. Statements :

All frogs are amphibians. Some turtles are amphibians. All turtles are reptiles.

Conclusion I: All frogs being turtles is a possibility.

Conclusion II: No reptile is a frong.

(a)  Either conclusion I or II is true

(b)  Both conclusions I and II are true.

(c)  Only conclusion II is true.

(d)  Neither conclusion I nor II is true.

(e)  Only conclusion I is true.

Answer: (e)

15. Statements:

Some plants are trees. All trees are weeds. All weeds are shrubs.

Conclusion I: All trees are shrubs.

Conclusion II: All shrubs being plants is a possibility.

(a)  Either conclusion I or II is true

(b)  Both conclusions I and II are true.

(c)  Only conclusion II is true.

(d)  Neither conclusion I nor II is true.

(e)  Only conclusion I is true.

Answer: (b)

16. It is possible to make only one meaningful English word with the third, fourth, seventh and the eighth letters of the word ‘VIDEOGRAPHY’, which would be the second letter of that word from the right end? If more than one such word can be formed, give ‘X’ as the answer. If no such word can be formed, give Z as your answer.

(a)  A

(b)  R

(c)  D

(d)  X

(e)  Z

Answer: (d)

Directions (Qs. 17 to 22) : In these questions, relationship between different elements is shown in the statements. The statements are followed by conclusions. Study the conclusions based on the given statements and select the appropriate answer.

17. Statements:

M ≥ O ≥ L ≥ T = E ≥ D

Conclusion I: D ≤ O

Conclusion II: M ≥ E

(a)  Either conclusion I or II is true

(b)  Both conclusions I and II are true.

(c)  Only conclusion II is true.

(d)  Neither conclusion I nor II is true.

(e)  Only conclusion I is true.

Answer: (b)

18. Statements:

B < C = D ≤ X ≤ Y < Z

Conclusion I: B < X

Conclusion II: Z ≤ C

(a)  Either conclusion I or II is true

(b)  Both conclusions I and II are true.

(c)  Only conclusion II is true.

(d)  Neither conclusion I nor II is true.

(e)  Only conclusion I is true.

Answer: (e)

19. Statement:

R < O ≤ L ≤ E; G = E ≥ S; P ≤ S

Conclusion I: R > P

Conclusion II: P ≤ E

(a)  Either conclusion I or II is true

(b)  Both conclusions I and II are true.

(c)  Only conclusion II is true.

(d)  Neither conclusion I nor II is true.

(e)  Only conclusion I is true.

Answer: (c)

20. Statement:

M ≥ O ≥ L ≥ T = E ≥ D

Conclusion I: T < O

Conclusion II: T = O

(a)  Either conclusion I or II is true

(b)  Both conclusions I and II are true.

(c)  Only conclusion II is true.

(d)  Neither conclusion I nor II is true.

(e)  Only conclusion I is true.

Answer: (a)

21. Statement:

S ≤ P ≤ A = R > E ≤ D

Conclusion I: A > D

Conclusion II: S ≤ E

(a)  Either conclusion I or II is true

(b)  Both conclusions I and II are true.

(c)  Only conclusion II is true.

(d)  Neither conclusion I nor II is true.

(e)  Only conclusion I is true.

Answer: (d)

22. Statement:

R < O ≤ L ? ≤ E; G = E ≥ S; P ≤ S

Conclusion I: O < G

Conclusion II: G = O

(a)  Either conclusion I or II is true

(b)  Both conclusions I and II are true.

(c)  Only conclusion II is true.

(d)  Neither conclusion I nor II is true.

(e)  Only conclusion I is true.

Answer: (a)

Directions (Qs. 23 & 24) : In these questions two statement A and B are given. Study both the statements carefully and select the best option gives the relation between two statements

23. Statement A:

In the past few y ears, enrolment in Government schools of country Matino has declined from 133.7 million to 121 million per year, while enrolment in private schools has increased from 51 million to 78 million per year.

Statement B:

Due to lack of funds, Government schools of Matino are unable to hire  properly trained teachers who would be able to provide good learning environment to the students.

(a)  Statement (B) is the cause and statement (A) is its effect.

(b)  Both the statements (A) and (B) are effects of some common cause.

(c)  Statement (A) is the cause and Statement (B) is its effect.

(d)  Both the statements (A) and (B) are effects of independent causes.

(e)  Both the statements (A) and (B) are independent causes.

Answer: (a)

24. Statement A:

The IT resources of company X are often measured by its employees which sometimes creates unwanted risk and liabilities for the company:

Statement B:

Company X will be monitoring online activities of its employees on official computers and may block content which it feels may adversely affect the productivity of the employees.

(a)  Statement (B) is the cause and statement (A) is its effect.

(b)  Both the statements (A) and (B) are effects of some common cause.

(c)  Statement (A) is the cause and Statement (B) is its effect.

(d)  Both the statements (A) and (B) are effects of independent causes.

(e)  Both the statements (A) and (B) are independent causes.

Answer: (c)

Directions (Qs. 25 to 30) : Study the following information carefully and answer the given questions.

   Eight persons – P, Q, R, S, T, U, V and W are sitting around a circular table with equal distance between each other, facing the centre, but not necessarily in the same order. Each one of them belongs to a different profession viz. Manager, Engineer, Chief, Pilot, Lawyer, Doctor, Architect and Teacher but not necessarily in the same order.

    V sits second to the right of the Manager. The Pilot and the Engineer are the immediate neighbours of V. R sits second to the right of T who is Lawyer. T is an immediate neighbour of the Pilot. Only one person sits between W and U. S sits third to the left of T. P sits exactly between U and S. The Architect sits second to the left of P. The Chef and the Teacher are immediate neighbours of the Architect. W is not Teacher.

25. Who among the following is a doctor?

(a)  R

(b)  V

(c)  Other than those given as options

(d)  W

(e)  P

Answer: (b)

26. Who sits exactly between R and the manager, when counted from the right of R?

(a)  W

(b)  T

(c)  Q

(d)  S

(e)  U

Answer: (e)

27. Which of the following statements is not true as per the given information?

(a)  R is a Chef.

(b)  P and V are immediate neighbours of S.

(c)  All the given statements are true.

(d)  Only three persons sit between U and Q.

(e)  S is an Engineer.

Answer: (a)

28. Four of the following five are alike in a certain way based on their positions in the arrangement and hence form a group. Which one does not belong to that group?

(a)  SU

(b)  RW

(c)  TQ

(d)  PU

(e)  VS

Answer: (a)

29. Who sits second to the right of Q?

(a)  S

(b)  P

(c)  Other than those given as options

(d)  W

(e)  U

Answer: (d)

30. If all the persons are made to sit in the alphabetical order in clockwise direction starting from P, the position of how many of them will remain unchanged (excluding P)?

(a)  Three

(b)  One

(c)  Two

(d)  Four

(e)  None

Answer: (a)

31. This question consists of an information followed by two statements numbered I and II.

The travel authorities of country Talong have eased the process of acquiring visa for the tourists.

Statement  I:

The economy of country Talong is more dependent on Tourism industry than any other

Statement II:

Students will be encouraged to go for higher studies to Talong.

Which of the given two statements can be inferred about the given information? (An inference is something which is not directly stated but can be inferred from the given facts.)

(a)  Neither I nor II can be inferred from the given information.

(b)  Both I and II can  be inferred from the given information.

(c)  Either I or II can  be inferred from the given information.

(d)  Only I can be inferred from t he given information.

(e)  Only II can be inferred from the given information.

Answer: (d)

Directions (Qs. 32 & 33) : Read the following information carefully and answer the given questions.

   In a class, Sameer’s rank is 10th from the top and Anjali’s rank is 4th from the top. Among girls, Anjali’s ran is 3rd from the top and 7th from the bottom. Among boys, Sameer’s rank is 5th from the top and 21st from the bottom.

32. What is the total number of students in that class?

(a)  43

(b)  30

(c)  34

(d)  39

(e)  36

Answer: (c)

33. How many boys and girls are in between Sameer and Anjali?

(a)  Four Boys, two Girls

(b)  Two  Boys, four Girls

(c)  Three Boys, two Girls

(d)  Two Boys, two Girls

(e)  Two Boys, three Girls

Answer: (c)

Directions (Qs. 34 to 39): Study the given information carefully to answer the given questions.

   Seven people – P, Q, R, S, T, U and V are sitting in a straight line with equal distance between each other, but not necessarily in the same order. Some of them are facing North and some are facing South.

   Only two people are sitting to the left of V. Only two people sit between V and Q. P sits second to the left of Q. The immediate neighbours of P face opposite directions (i.e. if one of the neighbours faces South then the other faces North and vice-versa). Only one person sits between P and R. U sits third to the left of R. S is not an immediate neighbour of Q. Both the immediate neighbours of R face the same direction (i.e. if one neighbour faces South then the other neighbour also faces South and vice-versa). P faces the same direction as that of R. T faces North. Q sits to the immediate left of T.

34. Who among the following sits exactly between V and the one who is sitting to the immediate left of Q?

(a)  P

(b)  R

(c)  Other than those given as options

(d)  T

(e)  U

Answer: (a)

35. Who amongst the following sits exactly in the middle of the line?

(a)  S

(b)  P

(c)  U

(d)  T

(e)  R

Answer: (b)

36. Which of the following statements is TRUE as per the given information?

(a)  S faces South.

(b)  V sits third to the left of Q.

(c)  None of the given options is true.

(d)  S sits exactly between R and P.

(e)  U sits to the immediate right of P.

Answer: (b)

37. Which of the following pairs represent the people sitting at the two extreme ends of the line?

(a)  QR

(b)  Other than those given as options

(c)  TU

(d)  SQ

(e)  ST

Answer: (e)

38. Who amongst the following sits second to the right of S?

(a)  U

(b)  V

(c)  Q

(d)  Other than those given as options

(e)  P

Answer: (b)

39. Which of the following pairs represent the immediate neighbours of Q?

(a)  TU

(b)  QP

(c)  PR

(d)  TV

(e)  VU

Answer: (a)

Direction (Qs. 40 to 44) : Read the given information and answer the given questions.

    Eight people S, T, U, V, W, X, Y and Z live on separate floors of an 8-floor building. Ground floor is numbered 1, first floor is numbered 2 and so an until the topmost floor is numbered 8.

• Y lives of floor numbered 1. Only two people live between Y and T.

• S lives immediately above X. S lives on an even numbered floor.

•  Only one person lives between X and W. X lives above W.

• Z lives on an odd-numbered floor above S.

• V does not live on the topmost floor.

40. Which of the following is true with respect to the given information?

(a)  Only one person lives between T and X.

(b)  Only two people live between T and U.

(c)  W lives immediately above Y.

(d)  Z lives on floor numbered 7.

(e)  U lives on an odd-numbered floor.

Answer: (d)

41. Who amongst the following lives between U and S?

(a)  Z

(b)  V

(c)  T

(d)  W

(e)  No one

Answer: (a)

42. Who among the following live on floor number 5?

(a)  U

(b)  X

(c)  V

(d)  T

(e)  Z

Answer: (b)

43. Who lives on the floor immediately below T?

(a)  S

(b)  Other than those given as options

(c)  Y

(d)  Z

(e)  W

Answer: (e)

44. How many people live between the floors on which Z and T live?

(a)  More than three

(b)  Three

(c)  No one

(d)  One

(e)  Two

Answer: (e)

Directions (Qs. 45 to 49): Study the information to answer the given questions.

    In a certain code language,

    ‘many residents of city’ is written as ‘ja nu pa la’

    ‘city with many choices’ is written as ‘ko la nu si’

     ‘choices made by residents’ is written as ‘pa mx ko tr’

     ‘made good choices here’ is written as ‘vk rpt r ko’

      (All the codes are two-letter codes only)

45. What is the code for ‘with’ in the given code language?

(a)  si

(b)  la

(c)  ko

(d)  tr

(e)  nu

Answer: (a)

46. What is the code for ‘made by resident’s in the given code language?

(a)  mx pa tr

(b)  ko mx pa

(c)  tr mx ko

(d)  la pa tr

(e)  mx tr vk

Answer: (a)

47. What is the code for ‘choices’ in the given code language?

(a)  ko

(b)  vk

(c)  la

(d)  nu

(e)  si

Answer: (a)

48. Which of the following may represent ‘here for good’ in the given code language?

(a)  la rp wq

(b)  xz vk si

(c)  rp uy vk

(d)  vk rp ja

(e)  vk rp la

Answer: (c)

49. What does the code ‘nu’ stand for in the given code language?

(a)  of

(b)  Either ‘city’ or ‘many’

(c)  with

(d)  Either ‘city’ or ‘good’

(e)  good

Answer: (b)

50. How many such pairs of letters are there in the word ‘MACHINERY’ each of which has as many letters between them in the word (in both the forward and backward directions), as they have between them in the English alphabetical series?

(a)  Three

(b)  Two

(c)  None

(d)  Five

(e)  More than three

Answer: (a)

QUANTITATIVE APTITUDE

Directions (Qs. 1 to 5) : In these questions two equations numbered I and II are given. You have to solve both the equations and mark the appropriate option.

1. I. 2x2 – x – 10 = 0

II. 2y2 – y – 21 = 0

(a)  x < y

(b)  x > y

(c)  x ≤ y

(d)  x ≥ y

(e)  Relationship between x and y cannot be established

Answer: (e)

2. I. 2x2 + 11x + 15 = 0

II. 4y2 + 22y + 24 = 0

(a)  x < y

(b)  x > y

(c)  x ≤ y

(d)  x ≥ y

(e)  Relationship between x and y cannot be established

Answer: (e)

3. I. 2x2 + 9x + 9 = 0

II. 2y2 + 17y + 36 = 0

(a)  x < y

(b)  x > y

(c)  x ≤ y

(d)  x ≥ y

(e)  Relationship between x and y cannot be established

Answer: (b)

4. I. 3x2 – 22x + 40 = 0

II. 2y2 – 19y + 44 = 0

(a)  x < y

(b)  x > y

(c)  x ≤ y

(d)  x ≥ y

(e)  Relationship between x and y cannot be established

Answer: (d)

5. I.3x2 – 16x + 21 = 0

II. 3y2 – 28y + 65 = 0

(a)  x < y

(b)  x > y

(c)  x ≤ y

(d)  x ≥ y

(e)  Relationship between x and y cannot be established

Answer: (a)

6. A merchant buys two items for Rs. 7500. One item he sells at a profit of 16% and the other item at 14% loss. In the deal the merchant makes neither any profit nor any loss. What is the difference between selling price of both the items? (in Rupees)

(a)  625

(b)  610

(c)  620

(d)  630

(e)  615

Answer: (c)

7. There was 120 litres of pure milk in a vessel. Some quantity of milk was taken out and replaced with 23 litres of water in such a way that the resultant ratio between the quantities of milk and water in the mixture was 4 : 1 respectively. Again 23 litres of the mixture was taken out and replaced with 27 litres of water. What is the respective ratio of milk and water in the resultant mixture?

(a)  58 : 37

(b)  116 : 69

(c)  69: 43

(d)  101 : 37

(e)  53 : 23

Answer: (*)

Directions (Qs. 8 to 12): Refer to the pie charts and answer the given questions.

8. If the respective ratio between total number of make professors and total number of female professors is 9 : 16 and the respective ratio between total number of male Assistant Professors and total number of female Assistant Professors is 9 : 11, what is the total number of female Professors and Assistant Professors teaching the given six subjects in University ‘X’?

(a)  63

(b)  90

(c)  77

(d)  73

(e)  87

Answer: (e)

9. What is the central angle corresponding to the total number of Professors and Assistant Professors teaching Hindi?

(a)  46.8°

(b)  50.4°

(c)  43.2°

(d)  39.6°

(e)  45.2°

Answer: (c)

10. What percent professors are teaching Psychology and Sociology together out of the total number of Professors and Assistant Professors teaching these two subjects together?

(a) 

(b) 

(c) 

(d) 

(e) 

Answer: (a)

11. The total number of Assistant Professors teaching Economics and English together are what percent more than the total number of Professors teaching these two subjects together?

(a)  138

(b)  145

(c)  130

(d)  142

(e)  140

Answer: (e)

12. What is the average number of Professors teaching Computer Science, Psychology, English and Sociology?

(a)  9

(b)  8

(c)  10

(d)  11

(e)  17

Answer: (a)

13. 15 years ago the average age of a family of four members was 40 years. Two children were born in that span of 15 years. The p resent average age of the family remained unchanged. Among the two children who were born in between the 15 years. If the older child at present is 8 years more than the younger one, what is the respective ratio between the present age of the older child and the present age of the younger child?

(a)  9 : 4

(b)  7 : 3

(c)  7 : 6

(d)  7 : 4

(e)  9 : 5

Answer: (b)

Directions (Qs. 14 to 18) : These questions consist of a question and two statements I and II given below it. You have to decide whether the data provided in the statements are sufficient to answer the question. Read both the statements and choose the appropriate option.

14. What is definitely the value of ‘x’?

(I) The value of  is equal to 0.

(II) The value of 3x – 2 ∙ 92x – 3 is equal to 95x – 19

(a)  The data either in statement I alone or in statement II alone are sufficient to answer the question.

(b)  The data in both the statements I and II together are not sufficient to answer the question.

(c)  The data in both the statements I and II together are necessary to answer the question.

(d)  The data in statement I alone are sufficient to answer the question while the data in statement II alone are not sufficient to answer the question.

(e)  The data in statement II alone are sufficient to answer the question while the data in statement I alone are not sufficient to answer the question.

Answer: (c)

15. An item was sold after giving a certain discount on the marked price. What was the percentage of discount given?

(I) The profit earned after giving the discounts is 44%. Had the percentage of discount been doubled the profit

(II) The cost price of the item is Rs. 400.

(a)  The data either in statement I alone or in statement II alone are sufficient to answer the question.

(b)  The data in both the statements I and II together are not sufficient to answer the question.

(c)  The data in both the statements I and II together are necessary to answer the question.

(d)  The data in statement I alone are sufficient to answer the question while the data in statement II alone are not sufficient to answer the question.

(e)  The data in statement II alone are sufficient to answer the question while the data in statement I alone are not sufficient to answer the question.

Answer: (c)

16. What is the respective ratio of initial investments of A and B?

(I) A started the business by investing a certain amount and he invested for the whole year. B joined A after 4 months from the start of the business and invested for the rest of the year.

(II) The initial investment of B was Rs. 10,800.

(a)  The data either in statement I alone or in statement II alone are sufficient to answer the question.

(b)  The data in both the statements I and II together are not sufficient to answer the question.

(c)  The data in both the statements I and II together are necessary to answer the question.

(d)  The data in statement I alone are sufficient to answer the question while the data in statement II alone are not sufficient to answer the question.

(e)  The data in statement II alone are sufficient to answer the question while the data in statement I alone are not sufficient to answer the question.

Answer: (c)

17. How much will the boat take to cover a distance of 63 km upstream?

(I) The difference between the time taken by the boat to travel from A to B (upstream) and time taken by it to travel from B to A (downstream) is 2 hours.

(II) The distance between A and B is 45 km and speed of the boat in still water is 12 kmph.

(a)  The data either in statement I alone or in statement II alone are sufficient to answer the question.

(b)  The data in both the statements I and II together are not sufficient to answer the question.

(c)  The data in both the statements I and II together are necessary to answer the question.

(d)  The data in statement I alone are sufficient to answer the question while the data in statement II alone are not sufficient to answer the question.

(e)  The data in statement II alone are sufficient to answer the question while the data in statement I alone are not sufficient to answer the question.

Answer: (c)

18. There are two cylindrical rollers- bigger and smaller roller. How many rotations will the bigger roller take to flatten a stretch of land (X)?

(I) The respective ratio of the radii of the bigger and the smaller roller is 7 : 3. Both the rollers are of the same length.

(II) The smaller roller takes 63 rotations to flatten the stretch of land (X).

(a)  The data either in statement I alone or in statement II alone are sufficient to answer the question.

(b)  The data in both the statements I and II together are not sufficient to answer the question.

(c)  The data in both the statements I and II together are necessary to answer the question.

(d)  The data in statement I alone are sufficient to answer the question while the data in statement II alone are not sufficient to answer the question.

(e)  The data in statement II alone are sufficient to answer the question while the data in statement I alone are not sufficient to answer the question.

Answer: (c)

19. A man takes  as long to row a distance upstream as to row the same distance downstream. What is the speed of the boat in still water  if it takes 3 hours to travel 38.4 km downstream? (in km/h)

(a)  11

(b)  10

(c)  12.4

(d)  12

(e)  14

Answer: (b)

20. In a company ‘XYZ’, the respective ratio between the total number of under-graduate employees and the total number of graduate employees is 13 : 23. The company has only two branches, one in Mumbai and one in Delhi. If the total number of under-graduate employees in Mumbai branch is 351, which is 30% of the total undergraduate employees in the company, what is the total number of graduate employee in the company?

(a)  2185

(b)  1955

(c)  2070

(d)  2360

(e)  2150

Answer: (c)

Directions (Qs. 21 to 25) : Which of the following will come in place of the question mar (?) in the given questions?

21. (30.06% of 560.14 + 53.02% of 1100) / 8 = ?

(a)  78

(b)  94

(c)  99

(d)  81

(e)  85

Answer: (b)

22. ? × 5 × 4.92 – 13.13 × 4.02 × 4 = 117

(a)  18

(b)  13

(c)  7

(d)  21

(e)  9

Answer: (b)

23. (9117.88 – 8021.85 + 903.92) × 12 = 1500 × ?

(a)  12

(b)  8

(c)  16

(d)  22

(e)  4

Answer: (c)

24. 

(a)  729

(b)  1849

(c)  2209

(d)  1369

(e)  1039

Answer: (e)

25. 95 × 5.04 + 237 – 302.11 = ?

(a)  5125

(b)  5000

(c)  5035

(d)  5005

(e)  5085

Answer: (c)

26. What is the difference between the total number of cars manufactured in countries A and B together and the total number of cars manufactured in country C and D together?

(a)  10000

(b)  12000

(c)  15000

(d)  6000

(e)  10500

Answer: (a)

27. Total number of cars sold in countries A and B together form what percent of the total cars manufactured by all the countries together in that particular quarter?

(a)  30

(b)  35

(c)  40

(d)  32

(e)  38

Answer: (b)

28. What is the total number of cars which remained unsold in the given quarter in the countries C & E together?

(a)  2400

(b)  3400

(c)  3650

(d)  3800

(e)  3600

Answer: (e)

29. What is the respective ratio between sales (in rupees) of the total number of cars sold in country D and the sales (in rupees) of the total number of cars sold in country E?

(a)  55 : 47

(b)  52 : 47

(c)  53 : 44

(d)  55 : 42

(e)  52 : 45

Answer: (d)

30. If the cost for manufacturing a car in country C is Rs. 4,50,000, what is the percent profit earned by the company, on which car sold, in the same company?

(a) 

(b) 

(c) 

(d) 

(e) 

Answer: (e)

31. Number of elephants in reserve A increased by what percent from 2001 to 2005?

(a)  56.5

(b)  62.5

(c)  68.5

(d)  54.5

(e)  58.5

Answer: (b)

32. What is the average number of elephants in reserve B during 2001, 2003, 2005 and 2006?

(a)  46

(b)  56

(c)  51

(d)  53

(e)  49

Answer: (b)

33. Out of the total number of elephants in reserves A and B together in 2002 only 40% were African elephants and out of the total number of elephants in reserves A and B together in 2004 only 65% were African elephants. What is the difference between total number of African elephants in reserves A and B together in 2002 and total number of African elephants in reserves A and B together in 2004?

(a)  37

(b)  47

(c)  43

(d)  41

(e)  55

Answer: (c)

34. What is the respective ratio between total number of elephants in reserve A in 2003 and 2006 together and total number of elephants in reserve B in 2005 and 2006 together?

(a)  17 : 18

(b)  23 : 24

(c)  11 : 14

(d)  9 : 14

(e)  11 : 12

Answer: (e)

35. Number of elephants in reserve A decreased by 25% from 2006 to 2007 and number of elephants in reserve B increased by 15% from 2006 to 2007. What was the total number of elephants in reserves A and B together in 2007?

(a)  172

(b)  164

(c)  158

(d)  174

(e)  166

Answer: (b)

36. There are two garbage disposal rectangular tanks, A and B with lengths 12 m and 15 m respectively in a square field. If the total area of the square field excluding the rectangular tanks is 360 sq. m. and the breadth of both the rectangular tanks is 1/3 of the side of the square field, what is the perimeter of the square field? (in m.)

(a)  92

(b)  84

(c)  96

(d)  78

(e)  72

Answer: (c)

37. Raman took a loan of Rs. 15,000 from Laxmna. It was agreed that for the first three years rate of interest charged would be at 8% simple interest per annum and at 10% compound interest (compounded annually) from the fourth year onwards. Ram did not pay anything until the end of the fifth year. How much would be repay if he clears the entire amount, only at the end of fifth year? (in Rupees)

(a)  22,506

(b)  22,105

(c)  22,900

(d)  22,500

(e)  22,450

Answer: (a)

Directions (Qs. 38 to 42) : Study the information carefully to answer these questions.

    Sidhartha has decided to start a new company ‘Sidhartha Travels’. He wants to buy some furniture – table, chair, air conditioners. He also wants to buy a few desktops which will include monitor, CPU, keyboard and mouse. The cost of each table is 13.5 times the cost of a mouse and the cost of a table. The cost of an air conditioner is 5 times the cost of a chair.

   The cost of a monitor is 20% more than the cost of a chair, a CPU costs Rs. 1500 more than a table and the keyboard costs 4 times a mouse. The cost of a mouse is Rs. 1000.

38. What will be the total cost of 1 table and 2 chairs together?

(a)  Rs. 23,200

(b)  Rs. 28,800

(c)  Rs. 29,700

(d)  Rs. 24,700

(e)  Rs. 21,300

Answer: (e)

39. What is the respective ratio between the cost of a CPU and the total cost of a mouse and a keyboard together?

(a)  3 : 2

(b)  4 : 1

(c)  3: 1

(d)  5 : 1

(e)  5 : 3

Answer: (c)

40. If the cost of a keyboard and mouse increases by 20% and 15%, respectively, what will be the total cost of a desktop (given that the cost of CPU and Monitor the same)?

(a)  Rs.36,930

(b)  Rs. 32,640

(c)  Rs. 34,753

(d)  Rs. 30,670

(e)  Rs. 35,425

Answer: (d)

41. Sidharth bought 1 table,, 1 chair and 1 desktop for himself. What was the total cost incurred by him?

(a)  Rs. 53,400

(b)  Rs. 51,460

(c)  Rs. 50,640

(d)  Rs. 55,440

(e)  Rs. 51,320

Answer: (e)

42. It was initially decided that 4 air conditioners will be installed but later only 3 air conditioners and a fan were installed. If the cost of a fan is 1/10th of the cost of air conditioner, what was the total cost incurred?

(a)  Rs. 1,29,400

(b)  Rs. 1,24,500

(c)  Rs. 1,25,550

(d)  Rs. 1,24,350

(e)  Rs. 1,25,600

Answer: (c)

43. Prem and Shyam decide to go on a trip to point Y on a particular day from point X. Prem leaves for point Y at 11:00 am, at speed of 72 km/hour. Shyam leaves for point Y at 11:30 same day as Prem left. At what speed should Shyam travel to catch up with Prem in 4 hours ? (in km/hour)

(a)  85

(b)  81

(c)  80

(d)  82

(e)  86

Answer: (b)

Directions (Qs. 44 to 48) : What will come in place of questions mark (?) in the given number series?

44. 2  12.9   9.5     14.6   7.8     ?

(a)  17.9

(b)  16.3

(c)  16.7

(d)  16.2

(e)  16.9

Answer: (b)

45. 8  5        9        5   61      ?

(a)  184

(b)  181.5

(c)  192.5

(d)  177.5

(e)  172.5

Answer: (d)

46. 7   9        24      84      ?        1810

(a)  336

(b)  356

(c)  348

(d)  340

(e)  352

Answer: (b)

47. 11   27      48      84      145    ?

(a)  241

(b)  239

(c)  263

(d)  257

(e)  229

Answer: (a)

48. 257, 226,   197,   170,   ?

(a)  170

(b)  180

(c)  153

(d)  165

(e)  145

Answer: (e)

49. ‘A’ began a small business with a certain amount of money. After four months from the start of the business. ‘B’ joins the business with an amount which is Rs. 6,000 less than ‘A’s’ initial investment. ‘C’ joins the business after seven months from the start of business with an amount with is Rs. 2,000 less than A’s initial investment. At the end of the year total investment reported was Rs. 1,42,000, what will be A’s share? (in Rupees)

(a)  48,000

(b)  46,500

(c)  46,000

(d)  42,000

(e)  48,500

Answer: (a)

50. If 6 men and 8 boys can do a piece of work in 10 days while 26 men and 48 boys can do the same in 2 days, how much time is taken by 15 men and 20 boys in doing the same type of work?

(a)  4 days

(b)  5 days

(c)  6 days

(d)  7 days

(e)  8 days

Answer: (a)

ENGLISH LANGUAGE

Directions (Qs. 1 to 15) : Read the following passage carefully and answer these questions. Certain words/phrases are given in bold to help you locate them while answering some of the questions

    In general, before the financial crisis of 2008, the financial sector the word over had been steadily liberalizing. Limits on foreign ownership of banks and on the kinds of transactions they were able to engage in were being lifted. Rich countries were deregulating faster than others. Banks were given greater leeway on how much capital they should hold and how much risk they should take on. But banks the world over did not maintain adequate capital cushions and balance sheets showed inflated profits. In 1999, America also repealed the Glass Staegall Act – a 1930s Depression era law separating investment and commercial banking without bothering about the threat to the economy.

   ‘Restrictions are a sign of backwardness’. But the resulting crisis of 2008 put an end to this belief. Banking supervisors in many developing countries said that tight regulations saved them from getting into trouble. Under the old rules supervisors were simply referees trying to ensure that the game was played fairly. Now regulators have gone from saying ‘tell me that all your payment systems work’ to saying ‘show me how your payment systems work’. Regulators are now tentatively stepping over a long standing divide between enforcing basic rules and playing a part in business decisions. This shift is particularly marked in Britain which once championed ‘light touch regulation’. This pre-crisis behaviour is being criticized as surrender to banks or as a self servicing device for attracting financial activity to Britain. In truth it was neither. It was the simple belief that markets are better than governments at allocating services. In America, too, regulators were reluctant to suppress innovation because they felt that “the self interest of lending institutions will be enough to ensure they did not all leap from the same tall building.”

   In rich countries, enthusiasm for prescriptive supervision depends on the degree of harm suffered during the banking crisis or to the threat from the failing banks to bring down their governments with them. But it is not easy to stop banks from making bad decisions. In the past, regulators left it to the market to judge the health of the banks. But clever, well-paid analyst failed to see the crisis coming. Now central bankers are expected to do a better job. One problem is that the rules and the laws are written with the benefit of hindsight. The good ideas that may have prevented the last crisis however can make regulators dangerously overconfident about being able to predict and prevent the next one. Also, if regulators underwrite certain strategies that seem safe such as lending to small business, they may encourage  banks to crowd into those lines of business. If enough banks pile into these markets, downturns in them can affect not just a few banks but the whole system. On the other hand, prescriptive supervision can stifle financial innovation and squeeze all appetite for risk out of the banking system. In Japan, a  banking crisis that started more than two decades ago still lingers on, in part because the country’s bankers have become gun shy and lend to buy government bonds rather than lend money or make foreign investments. Regulators are doing all they can to strike a balance and mitigate these risks.

1. Choose the word which is most nearly the SAME in meaning to the word LAST given in bold as used in the passage.

(a)  final

(b)  continue

(c)  first

(d)  recent

(e)  subsequent

Answer: (d)

2. Which of the following is the central idea of the passage?

(a)  Regulators are lazy and shirk their duty of protecting financial systems.

(b)  Banks should go back to traditional banking and abandon riskier options

(c)  Banks in developed countries have destroyed developing economies.

(d)  Today the task of financial regulation is tricky.

(e)  Financial systems have been damaged beyond repair.

Answer: (c)

3. Choose the word which is most nearly the SAME in meaning to the word CROWD given in bold as used in the passage.

(a)  multitude

(b)  flock

(c)  party

(d)  crew

(e)  not

Answer: (b)

4. Which of the following is/are the possible impact(s) of prescriptive supervision?

(A) Governments are likely to collapse as people are opposed to such measures.

(B) Many executives are likely to exploit the system.

(C) These measures could unintentionally prolong a crisis.

(a)  Only C

(b)  Only B and C

(c)  Only A & B

(d)  All A, B and C

(e)  Only B

Answer: (a)

5. What is the author’s view of central banks’ present efforts at regulation?

(a)  These are unnecessary and harmful to banks.

(b)  These are faulty as they encourage risky financial innovations.

(c)  To succeed these should be co-ordinated and uniform across counries.

(d)  The measures they prescribe have no loopholes.

(e)  They have done their best to effectively regulate.

Answer: (b)

6. Which of the following can be said about ‘light touch regulation’ adopted by Britain?

(a)  It gave Britain’s financial institutions very little autonomy.

(b)  It force banks to invest in government bonds.

(c)  It resulted in banks holding too much capital.

(d)  It encouraged financial activity in the country.

(e)  It stifled banks’ appetite for risk.

Answer: (d)

7. Choose the word which is mot nearly the SAME in meaning to the word LIMITS given in bold as used in the passage.

(a)  caps

(b)  rims

(c)  frames

(d)  frontiers

(e)  skirts

Answer: (a)

8. Which of the following is TRUE in the context of the passage?

(a)  The financial crisis of 2008 did not impact developing countries.

(b)  Markets can easily regulate themselves.

(c)  Developing economies should not allow foreign investment at present.

(d)  After the crisis, America’s central bank has imposed unnecessary regulations

(e)  None of the given statements is true in the context of the passage.

Answer: (a)

9. Why has the author cited the reference of repealing the Glass-Steagall Act?

(a)  To indicate that regulations were relaxed without appreciating the impact on the economy.

(b)  To criticize the backward restrictions that rich countries imposed on developing countries.

(c)  To show that the economy had not progressed much since the Depression.

(d)  To illustrate that only America could foresee the financial crisis.

(e)  To indicate the soundness of America’s financial system prior to Depression.

Answer: (a)

10. Which of the following difficulties is faced by regulators at present?

(a)  Banks lack the expertise to comply with norms.

(b)  Tremendous competition between local and foreign banks.

(c)  Striking a balance between protecting and stifling economy.

(d)  Unwillingness of government to bail out failing banks.

(e)  Lack of adequate manpower.

Answer: (c)

11. Which of the following approaches was adopted by the financial sector of rich world economies prior to the crisis?

(a)  Banks strictly adhere to outdated laws.

(b)  Banks maintained very large capital cushions.

(c)  They were innovative and took a lot of risks.

(d)  They withdrew investment from traditional banking.

(e)  Not clearly mentioned in the passage.

Answer: (c)

12. Choose the word which is most nearly the OPPOSITE in meaning to the word TENTATIVELY given in bold as used in the passage.

(a)  Temporary

(b)  Definite

(c)  Satisfactory

(d)  Ad hoc

(e)  Coercively

Answer: (d)

13. Which of the following is/are the consequence(s) of the crisis of 2008?

(A) Banks have become overconfident in their abilities to regulate themselves.

(B) Regulators have increased vigilance of financial systems.

(C) Economies are careful about foreign investment.

(a)  Only A

(b)  Only B & C

(c)  Only A & B

(d)  All A, B and C

(e)  Only A & C

Answer: (b)

14. Choose the word which is most nearly the OPPOSITE in meaning to the word FAILING given in bold as used in the passage.

(a)  increasing

(b)  passing

(c)  succesful

(d)  depleting

(e)  important

Answer: (c)

15. What does the author want to convey through the phrase ‘Under the old rules, supervisors were simply referees trying to ensure that the game was played fairly?

(a)  Regulators did not make mistakes while regulating financial markets.

(b)  Regulators were passive and did not intervene in the working of financial organizations in the past.

(c)  Regulators were concerned about the health of financial organizations.

(d)  Regulators used to cross the line and interfere in financial markets in the past.

(e)  Regulators devised many strict rules without taking into account the needs of financial systems.

Answer: (b)

Directions (Qs. 16 to 25): In these questions, some of the sentences have errors and some have none. Find out which part (a), (b), (c) or (d) has an error. Select that part as your answer. If none of the parts has an error, then mark (e) as your answer.

16. In the current year,/small lenders, who global banks have/ long sneered at, are performing/ more better than the giants.

(a)  In the current year

(b)  small lenders, who global banks have

(c)  long sneered at, are performing

(d)  more better than the giants.

(e)  No error

Answer: (b)

17. The Central Bank has cut/ its policy repo rate by fifty basis points/ but only a few banks has/ lowered their base lending rates.

(a)  The Central Bank has cut

(b)  its policy repo rate by fifty basis points

(c)  but only a few banks has

(d)  lowered their base lending rates.

(e)  No error

Answer: (c)

18. Apart of not speaking up enough,/ many professionals do not think about/ how asking different types of questions/ can lead to different outcomes.

(a)  Apart of not speaking up enough,

(b)  many professionals do not think about

(c)  how asking different types of questions

(d)  can lead to different outcomes.

(e)  No error

Answer: (a)

19. With the exception of/ music, no other industry/ has embraced globalization/ as keenly as the banking industry.

(a)  With the exception of

(b)  music, no other industry

(c)  has embraced globalization

(d)  as keenly as the banking industry.

(e)  No error

Answer: (e)

20. The bank’s stringent on boarding process/ for new executives ensures/ that they understand from other leaders/ what it takes to success.

(a)  The bank’s stringent on boarding process

(b)  for new executives ensures

(c)  that they understand from other leaders

(d)  what it takes to success.

(e)  No error

Answer: (a)

21. CEOs have a lot/ to worry about / but what are/ their greatest concerns?

(a)  CEOs have a lot

(b)  to worry about,

(c)  but what are

(d)  their greatest concerns?

(e)  No error

Answer: (e)

22. When business leaders/fail to decide/ an act wisely/ their companies suffers.

(a)  When business leaders

(b)  fail to decide

(c)  and act wisely,

(d)  their companies suffers.

(e)  No error

Answer: (d)

23. We are often blindly to/ the many forces/ that create and/ sustain our behaviours.

(a)  We are often blindly to

(b)  the many forces

(c)  that create and

(d)  sustain our behaviours

(e)  No error

Answer: (a)

24. Mediclaim policies usually/ reimburse certain/ medical expenses and / not the out-of-pocket expenses.

(a)  Medicalim policies usually

(b)  reimburse certain

(c)  medical expenses and

(d)  not the out-of-pocket expenses

(e)  No error

Answer: (e)

25. High performers show a stronger tendency/ to direct their own learning./ which may be one of the ways to predict/ who will be a high performer.

(a)  High performers show a stronger tendency

(b)  to direct their own learning,

(c)  which may be one of the ways to predict

(d)  who will be a high performer.

(e)  No error

Answer: (a)

Directions (Qs. 26 to 30) : In these questions, the given sentence has to blanks. Each blank indicates that something has been omitted. Choose the words that best fit the meaning of a sentence as a whole.

26. A new study has _________ the five most common health issues in men and how to ________ them.

(a)  said, arrest

(b)  stated, shrink

(c)  revealed, prevent

(d)  exposed, react

(e)  laid, restrict

Answer: (c)

27. ___________ up to the challenge of fighting drug addiction, children have ___________ up the task of spreading awareness against drugs.

(a)  Taking, set

(b)  Waking, sped

(c)  Rising, started

(d)  Seeing, taken

(e)  Fearing, given

Answer: (b)

28. The night fabric can do _________ for your frame and as a ________ your confidence.

(a)  wonders, consequence

(b)  marvels, side

(c)  good, process

(d)  remarkably, finish

(e)  superb, result

Answer: (a)

29. The wheat grains do not meet the __________standards_________ for procurement of food gains by the government agencies.

(a)  total, kept

(b)  most, meant

(c)  excellence, allowed

(d)  optimistic, placed

(e)  quality, prescribed

Answer: (e)

30. Students will be _________ on their reading and writing abilities through question papers _________ by the government.

(a)  judged, assorted

(b)  kept, set

(c)  tested, provided

(d)  evaluated, asked

(e)  calculated, assigned

Answer: (c)

Directions (Qs. 31 to 35) : Rearrange the following five sentences (A), (B), (C), (D) (E) & (F) in the proper sequence to form a meaningful paragraph, then answer the given question.

(A) But the conventional wisdom is wrong.

(B) The assumption is that planting trees and avoiding further deforestation provides a convenient carbon capture and storage facility on the land.

(C) Considering all the interactions, large-scale increases in forest cover can actually make global warming worse.

(D) Deforestation accounts for about 20 percent of global emissions of carbon dioxide.

(E) That is the conventional wisdom.

(F) In reality, the cycling of carbon, energy and water between the land and atmosphere is much m ore complex.

31. Which of the following should be second sentence after rearrangement?

(a)  A

(b)  B

(c)  C

(d)  D

(e)  F

Answer: (b)

32. Which of the following should be third sentence after rearrangement?

(a)  A

(b)  B

(c)  F

(d)  D

(e)  E

Answer: (e)

33. Which of the following should be fourth sentence after rearrangement?

(a)  A

(b)  B

(c)  C

(d)  E

(e)  D

Answer: (a)

34. Which of the following should  be first sentence after rearrangement?

(a)  A

(b)  B

(c)  C

(d)  D

(e)  E

Answer: (d)

35. Which of the following should be last (sixth) sentence after rearrangement?

(a)  A

(b)  F

(c)  D

(d)  C

(e)  B

Answer: (b)

Directions (Qs. 36 to 40) : Which of the phrases given below the sentence should replace the word/phrase given in bold in the sentence to make it grammatically correct? If the sentence is correct as it is given and no correction is required, select ‘No correction required’ as your answer.

36. Whether people change or not is largely determines by ‘why’ they change.

(a)  large determinant of

(b)  largely determined by

(c)  large determination by

(d)  larger determinant for

(e)  No correction required

Answer: (b)

37. With the right mentoring at the top, everyone stands to gain.

(a)  for gain

(b)  to gaining

(c)  of gained

(d)  to gained

(e)  No correction required

Answer: (e)

38. The best leaders know how to kept moving forward even in ambiguous situations.

(a)  know how to keep

(b)  knows how to keep

(c)  know how for keeping

(d)  knowing what to keep

(e)  No correction required

Answer: (a)

39. Asking questions helps us better understand what has been say.

(a)  has being said

(b)  has been said

(c)  have been say

(d)  has been saying

(e)  No correction required.

Answer: (a)

40. A high performer is four hundred times much productivity than the average performer.

(a)  more produce

(b)  productive more

(c)  more productive

(d)  much products

(e)  No correction required

Answer: (c)

Directions (Qs. 41 to 50) : In the following passage, there are blanks, each of which has been numbered. Against each, five words are suggested, one of which fits the blank appropriately. Find out the appropriate word in each case.

   Visual experiences can …..(41)…. children, teenagers and even adults learn and absorb more due to its highly stimulating and ….(42)….. engaging impact. it is for this reason that we are seeing an increase in schools across the globe ….(43)…. content provider programmes into their class curriculum to ….(44)…. lessons through video. Visual excursions and school collaborations are ….(45)…. by advances in high definition video, high fidelity audio and content sharing, allowing students to experience a richer and m ore stimulating learning transported students to excursions in …(46)…, now face increased transportation costs, higher insurance premiums, attendance costs for the families and strict duty of care policies for students while ….(47)… school property. Virtual excursions …(48)… students to improve their presentation, research, learning and speaking skills while they engage in a live learning session.

     Students also now have the ability to meet peers from many cultures, speak to subject-matter …(49)… like scientists or authors’ practice a foreign language with students from another country, and learn about global issues from the ….(50)… of their own classrooms.

41.

(a)  help

(b)  aiding

(c)  prescribe

(d)  feature

(e)  present

Answer: (a)

42.

(a)  plus

(b)  lonely

(c)  ably

(d)  many

(e)  deeply

Answer: (e)

43.

(a)  incorporating

(b)  pressing

(c)  following

(d)  parting

(e)  leaving

Answer: (a)

44.

(a)  make

(b)  demand

(c)  impart

(d)  vision

(e)  need

Answer: (c)

45.

(a)  dissolved

(b)  enhanced

(c)  measured

(d)  failed

(e)  blasted

Answer: (c)

46.

(a)  deed

(b)  total

(c)  parent

(d)  person

(e)  lieu

Answer: (d)

47.

(a)  involving

(b)  saving

(c)  away

(d)  off

(e)  vacating

Answer: (b)

48.

(a)  let

(b)  enable

(c)  present

(d)  pressure

(e)  collect

Answer: (b)

49.

(a)  clauses

(b)  dictionaries

(c)  books

(d)  experts

(e)  partners

Answer: (d)

50.

(a)  vacancy

(b)  availability

(c)  safety

(d)  comfortable

(e)  gap

Answer: (c)

Bank of Baroda Probationary Officers Examination Held on 25-9-2016 Quantitative Aptitude Question With Answer Key

Bank of Baroda Probationary Officers Examination Held on 25-9-2016 Quantitative Aptitude
Bank of Baroda Probationary Officers Examination Held on 25-9-2016 Quantitative Aptitude Question With Answer Key

 Bank of Baroda Probationary Officers Examination-2016

Held in 25-9-2016

Quantitative Aptitude

Direction- What will come in place of question-mark (?) in the given number series ?

1. 4 1  26  57  120

(A)  247

(B)  251

(C)  187

(D)  236

(E)  215

Answer: (A)

2. 50 81  121  172  236  ?

(A)  392

(B)  309

(C)  361

(D)  379

(E)  315

Answer: (E)

3. 84 4  85.6  80.8  ?  79.2

(A)  72.8

(B)  86.4

(C)  88.8

(D)  87.2

(E)  82.4

Answer: (D)

4. 900 448  220  104  44  ?

(A)  8

(B)  16

(C)  24

(D)  42

(E)  12

Answer: (E)

5. 60 ?  120  80  40  16

(A)  90

(B)  110

(C)  120

(D)  70

(E)  100

Answer: (C)

6. 2 3  7  29  ?  3729

(A)  233

(B)  225

(C)  227

(D)  331

(E)  191

Answer: (A)

7. Raju’s age four years hence is equal to Simi’s present age. At present Ganga is years older to Raju. The respective ratio between Simi’s age eight years hence and Ganga’s age two years hence is 10 : 9. What is the Raju’s present age ?

(A)  24 years

(B)  18 years

(C)  32 years

(D)  20 years

(E)  28 years

Answer: (E)

8. Five years ago, Somi’s age at that time was  of Amit’s age at that time. The respective ratio between Amit’s age six years hence and Somi’s age twelve years hence, will be 7 : 4 what was Somi’s age three years ago ? (In years)

(A)  13

(B)  29

(C)  17

(D)  25

(E)  23

Answer: (C)

9. A n umber ‘n’ is selected randomly from a set of numbers 1, 2, 5. Another number ‘m’ selected randomly from a set of numbers 2, 3, 7. What is the probability that product of m and n (mn) is less than 12 ?

(A)  2/3

(B)  5/9

(C)  1/3

(D)  1/8

(E)  1/6

Answer: (A)

10. The respective ratio between the monthly salary of Ram and that of Shyam is 5 : 6. Ram and Shyam, both are 40% and 25% out of their respective monthly salary Ram invests  of h is savings in LIC and Shyam invests  of her savings in LIC. If Ram invests Rs. 1750 more than Shyam in LIC, What is Ram’s monthly salary ?

(A)  Rs. 20,000

(B)  Rs.25,000

(C)  Rs. 40,000

(D)  Rs. 15,000

(E)  Rs. 30,000

Answer: (B)

Directions – Refer to the pie charges and answer the given question.

11. 1/3rd of the number of students (both male and female) in institute C are Science graduates. If the number of male science graduate students in institute C is 32, what percentage of female students in institute C are science graduates ?

(A)  45

(B)  35

(C)  30

(D)  40

(E)  50

Answer: (D)

12. Number of students (both male & female) in institute A increased by 50% from 2013 to 2014. If the respective ratio of number of male and female students in 2014 in institute A was 4 : 3, what was the number of female students in institute A in 2014 ?

(A)  183

(B)  195

(C)  171

(D)  189

(E)  135

Answer: (D)

13. What is the central angle corresponding to the number of students (both male & female) in institute D ?

(A)  68.4°

(B)  65.2°

(C)  71.3°

(D)  61.2°

(E)  67.6°

Answer: (A)

14. What is the average number of male students in institutes A, D and E ?

(A)  242

(B)  282

(C)  248

(D)  246

(E)  238

Answer: (C)

15. Number of female students in institute B is what per cent more the number of male students in institute C?

(A)  30

(B)  29

(C)  32

(D)  42

(E)  36

Answer: (C)

16. What is the difference between total number of female students in institutes A and E together and number of students (both male & female) in institute B?

(A)  215

(B)  213

(C)  306

(D)  203

(E)  217

Answer: (C)

Directions-What approximate value will come in place of question mark (?) in the given question ? (You are not expected to calculate the exact value)

17. 87 + 24.11 ÷ 3.99 × 249.899 ÷ 12= ?

(A)  405

(B)  300

(C)  200

(D)  385

(E)  460

Answer: (B)

18. 

(A)  255

(B)  395

(C)  175

(D)  25

(E)  295

Answer: (C)

19. 96 + 8420.02 ÷ (5.001 × 39.999) = ?

(A)  540

(B)  580

(C)  500

(D)  460

(E)  620

Answer: (A)

20. 

(A)  4

(B)  5

(C)  2

(D)  8

(E)  6

Answer: (A)

21. 

(A)  86

(B)  28

(C)  42

(D)  4

(E)  54

Answer: (D)

22. 

(A)  224

(B)  120

(C)  169

(D)  88

(E)  146

Answer: (E)

23. 15 men can finish a piece of work in 8 days. 18 women can finish the piece of work in 12 days. 6 men and 6 women started working together and worked for 10 days. If the remaining work needed to be finished in 2 days, how many men should be added to the existing number of workers ?

(A)  2

(B)  5

(C)  3

(D)  4

(E)  6

Answer: (D)

24. The length of train A and B are 300 m and 360 m respectively. When the trains are travelling in opposite directions they take 12 sec to cross each other (from the moment they meet.) When they are travelling in same direction they take 2 min 12 sect to cross each other (from the moment they meet). What is the speed of the slower train ? (in km/hr).

(A)  117

(B)  90

(C)  72

(D)  126

(E)  130

Answer: (B)

25. A, B and C invest in a business. B invests an amount which was twice of that invested b y A. C invests an amount which was thrice of that invested by A. At the end of eight months from the start of the business, A and B both double their respective investments. If at the end of the year, C’s share out of the total annual profit earned was Rs. 13,500. What was the total profit earned that year ?

(A)  Rs. 31,500

(B)  Rs. 28,500

(C)  Rs. 32,500

(D)  Rs. 45,000

(E)  Rs. 32,000

Answer: (A)

26. Every year Meenakshi invests 20% of her annual salary in PPF and spends 30% of her annual salary on paying rent. Every month, she spends 20% and 15% of the monthly salary on groceries and travelling respectively. After paying the mentioned monthly expenses (including rent), she puts the remaining amount in the bank as savings. In a year, she is able to put Rs. 1,79,760 in the bank as savings. If she invests in PPF from her annual savings at once, what is the amount that she invests in PPF ?

(A)  Rs. 1,03,600

(B)  Rs. 1,04,800

(C)  Rs. 1,02,720

(D)  Rs. 1,05,600

(E)  Rs. 1,02,800

Answer: (C)

Direction- Refer to the answer the answer the given question.

27. The number of people who watched Movie B on screen-5 was what per cent less than those who watched the same movie on scree-3 ? (Rounded off to two digits after decimal)

(A)  37.83

(B)  39.55

(C)  35.33

(D)  38.99

(E)  33.33

Answer: (E)

28. What is the difference between the total number of people who watched movie A on Screens-3 and 4 together and those who watched Movie B on the same screens together ?

(A)  50

(B)  60

(C)  90

(D)  80

(E)  70

Answer: (C)

29. What is the average number of people who watched Movie B on screens-2, 3 and 4 ?

(A)  325

(B)  312

(C)  315

(D)  320

(E)  310

Answer: (D)

30. On Sunday, the number of people who watched Movie A on screens-1 and 2 increased by 12% and 10% respectively as compared to those who watched the same movie on the respective screens on previous day. What is the respective ratio between the number of people who watched movie A on Screens-1 and 2 on Sunday ?

(A)  14 : 11

(B)  23 : 22

(C)  14 : 9

(D)  15 : 11

(E)  21 : 11

Answer: (A)

31. The number of people who watched both the given movies together o Screen-5 is what per cent of the total number of people who watched both the given movies together on Screen-1 ?

(A)  52

(B)  44

(C)  36

(D)  48

(E)  32

Answer: (D)

32. Equal number of males watched Movies A and B on screen-2. If the respective ratio between the Number of female who watches movie A on Screen-2 and those who watched Movie B on the same screen was 7 : 12, what is the number of male who watched Movie-A on screen 2 ?

(A)  200

(B)  140

(C)  180

(D)  120

(E)  160

Answer: (C)

Directions- Study the following information carefully and answer the given questions-

     In a conference, total 240 students are there. Out of them 25% know Latin, 50% know French and 62.5% know English. 10 students know all there language. 25/3% of students know Latin and French but does not know English and the same number of students know Latin and English but does not know French.

33. How many students are there who know both English and French but do not know Latin ?

(A)  10

(B)  20

(C)  30

(D)  40

(E)  50

Answer: (C)

34. How many students are there who know only French ?

(A)  60

(B)  90

(C)  120

(D)  80

(E)  75

Answer: (A)

35. How many students are there who know exactly one language ?

(A)  240

(B)  220

(C)  200

(D)  180

(E)  160

Answer: (E)

36. How many students are thee who know exactly two languages ?

(A)  60

(B)  70

(C)  80

(D)  90

(E)  120

Answer: (B)

37. How many students known at most two languages ?

(A)  60

(B)  70

(C)  220

(D)  230

(E)  240

Answer: (D)

38. 

(A)  x < y

(B)  Relationship between x and y cannot be established

(C)  x ≥ y

(D)  x . y

(E)  x ≤ y

Answer: (D)

39. 

(A)  x < y

(B)  Relationship between x and cannot be established

(C)  x ≥ y

(D)  x > y

(E)  x ≤ y

Answer: (D)

40. I. (2x2 – 11x + 5) – (x2 – 19x – 7) = 0

    II. (y2 – 5y + 3) + (6y + 3 – 2y2) = 0

(A)  x < y

(B)  Relationship between x and y cannot be established

(C)  x ≥ y

(D)  x > y

(E)  x ≤ y

Answer: (E)

41. I. x = 4y – 9

    II. y = −5 – x

(A)  x  < y

(B)  Relationship between x and y cannot be established

(C)  x ≥ y

(D)  x > y

(E)  x ≤ y

Answer: (A)

42. I. 5x2 – 8x + 13 = 0

    II. 6y2 + 13y – 15 = 0

(A)  x < y

(B)  Relationship between x and y cannot be established

(C)  x ≥ y

(D)  x > y

(E)  x ≤ y    

 

Answer: (B)

Directions- Study the table and answer the given question :

43. In the month of July, the number of people travelling along Route R is 20% more than that of the same route in the previous months approximately by what per cent is the number of people travelling along route R in the month of July more than the number of people travelling along Route S in the month of February?

(A)  57

(B)  38

(C)  78

(D)  138

(E)  62

 

Answer: (B)

44. What was the average number of people who travelled along route P in the months of January, March, April and June (in Lakh) ?

(A)  2.4

(B)  2.6

(C)  2.2

(D)  1.8

(E)  1.6

 

Answer: (A)

45. What was the percentage reduction in the number of people travelling along Route T in the month of April when compared to that along the same route in the month January?

(A)  15

(B)  25

(C)  20

(D)  32

(E)  47

 

Answer: (D)

46. What is the difference between the total number of people who travelled along route R during the months of February and May together and the total number of people of who travelled along Route Q during the months of January and June together ?

(A)  100000

(B)  110000

(C)  150000

(D)  120000

(E)  135000

 

Answer: (D)

47. What is the difference between the highest and the lowest number of people who travelled along Route S during all the given months ?

(A)  30000

(B)  10000

(C)  50000

(D)  60000

(E)  40000

 

Answer: (E)

48. What is the respective ratio between the total number of people who travelled along Route S in the months of February, March and June together and the total number of people who travelled along Route Q in the months of January, March and April together ?

(A)  13 : 9

(B)  11 : 7

(C)  9 : 5

(D)  5 : 4

(E)  None of these

 

Answer: (A)

49. 1/3rd of a right circular cylindrical cistern, whose diameter and height are 28 cm and 12 cm respectively, is full. An iron cuboid whose dimensions are 12 cm, 10.5 cm and 8 cm was immersed into it (the block was totally submerged). how many cubic centrimetre of water need to be added to the custom to make it completely full ?

(A)  3750

(B)  3800

(C)  3690

(D)  3920

(E)  3630

 

Answer: (D)

50. In a class, three students weighing 75 kg, 71 kg and 63 kg respectively leave the class and are replaced by three students weighing 47 kg, 49 kg and 53 kg respectively. As a result, the average weight of the class decreased by 1.25 kg. How many students are there in the class ?

(A)  48

(B)  36

(C)  40

(D)  44

(E)  52

 

Answer: (A)

Bank of Baroda Probationary Officers Examination Held on 25-9-2016 Reasoning Question With Answer Key

Bank of Baroda Probationary Officers Examination Held on 25-9-2016 Reasoning
Bank of Baroda Probationary Officers Examination Held on 25-9-2016 Reasoning Question With Answer Key

Bank of Baroda Probationary Officers Examination-2016

Held On 25-9-2016

Reasoning

Directions- (Q. 1-5) Study the following information to answer the given question-

   Sven stores viz. A, B, C, D, E, F and G sold different number TV’s of a particular brand viz. 4, 6, 9, 12, 15, 18 and 27 on seven different days of the same week starting from Monday and ending on Sunday, but not necessarily in the same order.

• A sold the TV’s on one of the days after Friday. Only three stores sold TV’s between A and the store which sold 12 TV’s. The store which sold TV’s on the day immediately after the store which sold 12 TV’s sold more than 12 TV’s but not odd number of TV’s.

• Only two stores sold TV’s between F and the store which sold 4 TV’s. The store which sold 4 TV’s sold these before F. 4TV’s were not sold on Monday.

• Only three stores sold TV’s between F and D. The difference between the number of TV’s sold on Wednesday and Saturday is more than 10.

• B sold the TV’s on a day immediately before the day on which 9 TV’s were sold. 9 TV’s were not sold on Friday.

• Only two stores sold TV’s between B and G.

• Difference between the number of TV’s sold by G and D is less than 5. C did not sell the least number of TV’s.

1. In which of the following pairs did both the stores sell odd number of TV’s?

(A)  D, E

(B)  A, B

(C)  E, F

(D)  F, G

(E)  C, D

Answer: (B)

2. Which of the following stores sold 18 TV’s?

(A)  F

(B)  A

(C)  C

(D)  G

(E)  E

Answer: (C)

3. Based on the given arrangement, which of the following is true?

(A)  D sold the TV’s on Wednesday

(B)  Only one store sold TV’s between C and F

(C)  A sold 12 TV’s on Sunday

(D)  C sold 18 TV’s on Thursday

(E)  None of the given options is true

Answer: (D)

4. How many stores sold less number of TV’s than B?

(A)  Five

(B)  Three

(C)  Six

(D)  Two

(E)  One

Answer: (C)

5. What is the difference between the number of TV’s sold on Monday and Friday?

(A)  9

(B)  12

(C)  18

(D)  3

(E)  11

Answer: (A)

Directions- (Q. 6-10) Study the given information carefully and answer the given question-

   When a word and number arrangement machine is given an input line of words and numbers, it arranges them following a particular rule. The following is an illustration of input and arrangement :

    (All the numbers are two digit numbers)

Input : 48 height marker 13 92 kneel school 29 65 12.

Step I : barriers 48 height marker 92 kneel school 29 65 12.

Step II: 28 barriers 48 marker 92 kneel school 65 12 heights.

Step III : kneels 28 barriers marker 92 school 65 12 heights 47.

Step IV : 64 kneels 287 barriers 92 school 12 heights 47 markers.

Step V : schools 64 kneels 28 barriers 12 h eights 47 markers 91.

    Step V is the lasts step of the above arrangement as the intended output of arrangement is obtained.

    As per the rules followed in the given steps, find the appropriate steps for the given input.

    Input : 84 layer 97 packet 51 damage narrow 75 32 table.

6. Which of the following represents the sum of the fourth element from the left end in step V and the second element from the right end in step II?

(A)  105

(B)  125

(C)  114

(D)  147

(E)  81

Answer: (E)

7. ‘damages’ is related to ‘97’ in step III in the same way as ‘narrow’ is related to ‘table’ in step I. Following the same pattern to which element is 31 related to in step IV?

(A)  50

(B)  74

(C)  packets

(D)  damages

(E)  97

Answer: (B)

8. Which of the following represents the element that is fifth to the left of ‘layers’ in the last step?

(A)  narrows

(B)  50

(C)  packets

(D)  83

(E)  table

Answer: (D)

9. In step IV, how many elements are there between ‘50’ and the fourth element from the right end?

(A)  More than three

(B)  None

(C)  Three

(D)  Two

(E)  One

Answer: (C)

10. Which of the following is the third to the left of the ninth element from the left end of step II?

(A)  damages

(B)  75

(C)  narrow

(D)  97

(E)  packet

Answer: (C)

 Directions – (Q. 11-13) Study the given information carefully to answer the given question –

   Roshni starts  walking towards north from Point Q. She walks for 15 m and reaches Point R. She takes left turn and walks for 8 m to reach Point M. From Point M she takes a right turn and walks for 4 m she takes a right turn and walks for 4 m. She then takes a left turn and reaches Point K after walking for 5 m.

    Azhar is standing 8 m west of Point M. He walks 6 m towards north. He takes a right turn and walk for 3 m to reach Point D. Point S is 11 m south of Point D.

11. In which direction is Point K with respect to Roshni’s starting position?

(A)  North

(B)  South

(C)  North-west

(D)  West

(E)  South-east

Answer: (C)

12. If Point C is 13 m west of point Q, how far and in which direction is Point S with respect to Point C?

(A)  15 m to the East

(B)  10 m to the West

(C)  10 m to the North

(D)  15 m to the North

(E)  6 m to the South

Answer: (C)

13. How far is Point R from Azhar’s starting position?

(A)  10 m

 (B) 18 m

(C)  12 m

(D)  16 m

(E)  13 m

Answer: (D)

Directions- (Q. 14-18) Study the given information carefully to answer the given question-

     Eight people M, N, O, P, Q, R, S and T are sitting around a circular table facing the centre with equal distance between each other, but not necessarily in the same order. Each person is dressed as a different cartoon character-Scrooge, Dexter, Minion, Nobita, Oswald, Popeye, Yogi and Simpson, but not necessarily in the same order S sits second to the left of the one who is dressed as Yogi. Only two people sit between the one who is dressed as Yogi and the one who is dressed as Scrooge. O sits to the immediate left of the one who is dressed as Scrooge. Only one person sits between O and the one who is dressed as Dexter. S neither is dressed as Dexter nor Simpson. N sits third to the left of the one who is dressed as Dexter nor Simpson. N sits third to the left of the one who is dressed as Simpson. N is neither dressed as Dexter nor Yogi. M sits to the immediate left of the one who is dressed as Simpson. Only three people sit between M and the one who is dressed as Nobita. Q sits second to the left of the one who is dressed as Nobita. T and P are immediate neighbours of each other. T is not dressed as Nobita. The one who is dressed as Oswald sits to the immediate right of P. M is not dressed right of P. M is not dressed as Popeye.

14. Which of the following is true with respect to the given arrangement?

(A)  M is dressed as Minion

(B)  Only two people sit between R and the one who is dressed as Dexter

(C)  The one who is dressed as Popeye sits to the immediate right of O

(D)  None of the given options is true

(E)  The one who is dressed as Yogi and Q are immediate neighbours of each other

Answer: (A)

15. How many people sit between M and the one who is dressed as Scrooge, when counted from the right of M?

(A)  Four

(B)  One

(C)  Two

(D)  More than four

(E)  None

Answer: (B)

16. Who amongst the following is dressed as Oswald?

(A)  T

(B)  R

(C)  Q

(D)  M

(E)  S

Answer: (E)

17. Who sits third to the right of one who is dressed as Minion?

(A)  The one who is dressed s Scrooge

(B)  M

(C)  The one who is dressed as Oswald

(D)  Q

(E)  T

Answer: (E)

18. In the given arrangement, T is related to the one who is dressed as Popeye in the same way as O is related to the one who is dressed as Nobita. Following the same pattern, to who is N related?

(A)  The one is dressed as Dexter

(B)  The one who is dressed as Oswald

(C)  The one who is dressed as Simpson

(D)  The one who is dressed as Minion

(E)  The one who is dressed as Yogi

Answer: (C)

19. Read the following information and answer the given question-

The Handloom industry of Country X provides employment to almost one-third of its population. However, the industry’s contribution to the economy is very less as compared to other industries. Recently, the Government took series of initiatives like skill development, reduced cost of finance etc. to help the industry grow. Which of the following can be a short term effect of the initiatives taken by the Government of Country X?

(A)  Every person engaged in the Handloom industry will have higher skills due to the skill development programmes of the Government

(B)  Handloom industry’s contribution to the economy of Country X will be highest in the next financial year

(C)  Two-third of the population will start working in the Handloom industry of country X

(D)  Ministry of people in Country X will prefer handloom products over other products

(E)  Handloom products development by handloom industry of country X will improve atleast to some extent

Answer: (B)

20. How many such pairs of letters are there in the word BACTERIA each of the which has as many letters between them (both in forward and backward direction) in the word as they have in the English alphabetical series?

(A)  More than three

(B)  None

(C)  Two

(D)  Three

(E)  One

Answer: (D)

Directions – (Q. 21-25) Study the following information to answer the given question-

    Eight friends, A, B, C, D, E, F, G and H are seated in a straight line with equal distance between each other, but not necessarily in the same order. Some them are facing north while some are facing south.

• B sits fifth to the left of G. Neither B nor G sits at an extreme end of the line.

• A sits second to the right of B.

• Only two people sit between A and F.

• D sits third to the left of H. D is not an immediate neighbour of A. Neither H nor E is an immediate neighbour of B.

• Immediate neighbours of A face opposite direction (i.e., if one neighbours faces north then the other faces south and vice-versa.)

• E faces north. Immediate neighbours of B face same direction (i.e., if one neighbours faces north then the other also faces north and vice-versa.)

• Both A and B face a direction opposite to that of F. (i.e., if F facs north then both A and D face south and vice-versa.)

• C and E face opposite directions (i.e., if C faces north then E faces south and vice-versa.)

21. What is the position of F with respect to B?

(A)  Second to the left

(B)  Third to the right

(C)  Immediate left

(D)  Fourth to the left

(E)  Immediate right

Answer: (C)

22. Which of the following pairs represents persons seated at the two extreme ends of the line?

(A)  E, H

(B)  F, D

(C)  F, E

(D)  C, A

(E)  D, C

Answer: (B)

23. Four of the given five are alike in a certain way based on the given arrangement and hence form a group. Which of them does not belong to that group?

(A)  FC

(B)  BF

(C)  EH

(D)  DE

(E)  AB

Answer: (E)

24. Based on the given arrangement, which of the following is true with respect to C?

(A)  B sits to the immediate left of C

(B)  H is an immediate neighbour of C

(C)  Only two people sit between C and F

(D)  None of the given options is true

(E)  E sits third to the left of C

Answer: (A)

25. Which of the following sits exactly between G and A?

(A)  CF

(B)  EH

(C)  FB

(D)  HC

(E)  BE

Answer: (B)

26. This question consist of a statement followed by two courses of action I and II given below it. A course of action is an administrative decision to be taken for improvement, follow-up or further action in regard to the problem, policy etc. You have to assume everything in the statement to be true and decide which of the suggested course of action’s logically follows from the given statement.

Statement : ‘Global Girl’, a famous store exclusively for woman apparel. hired new designers 16 months ago for a contract of minimum 3 years. Though ‘Global Girl’ is dissatisfied with the repetitive designs, they cannot fire the designers owing to the contract.

Course of Actions :

A renowned designer should be hired by ‘Global Girl’ who can guide their designers with new designs as per the current trends.

The salaries of the designers should be considerable reduced and moreover, they should be issued a letter stating their disappointment with the designers.

(A)  Either I nor II follows

(B)  Both I and II follow

(C)  Only II follows

(D)  Only I follows

(E)  Neither I nor II follows

Answer: (D)

27. The number of talent based reality shows being broadcasted for entertainment of people have geared up in the past few years.

Which of the following may not be a reason for the increased number of talent based reality shows?

(A)  The talent based reality shows act as a source of knowledge as well as entertainment for the common masses

(B)  As compared to melodramatic shows the concept in talent based reality shows is not repetitive in nature

(C)  The talent based reality shows help in building competitive spirit amongst the viewers as well as the participants

(D)  The cost of making reality shows required huge investment in changing props and sets

(E)  The talent based reality shows help in building an emotional connect between the participants and the viewers

Answer: (D)

28. Which of the following expression will be definitely false if he given expressions ‘D > I ≤ F = C > U ≥ L’: ‘T > C’ are definitely true?

(A)  L < F

(B)  D < T

(C)  C ≥ I

(D)  C < D

(E)  T > L

Answer: (D)

29. Statement : “Company X has approached the a proposal to set up an emergency unit in the only hospital of village Y.”

Which of the following could possibly lead the government to accept the proposal of company X?

(a)   The emergency cases that come to the hospital of village Y are not much in number.

(b)   At present there is no emergency unit in the hospital of village Y where people can visit in case of emergencies.

(c)   The emergency unit in one of the nearby hospitals, where people of village Y visit is overcrowded most of the times.

(A)  Both (b) and (c)

(B)  Only (b)

(C)  Both (a) and (b)

(D)  All (a), (b) and (c)

(E)  Only (c)

Answer: (A)

30. Which of the following symbols should replace the question mark in the given expression in order to make the expressions ‘V ≥ Y’ as well as ‘I < L’ definitely true ?

L > E = V ? I ≥ N = G ≥ Y

(A)  ≤

(B)  Either ‘≥’ or ‘=’

(C)  Either ‘=’ or ‘≤’

(D)  >

(E)  <

Answer: (B)

Directions-(Q. 31-36) In this question three statements followed by two conclusions I and I have been given. You  have to take the given statements to be true even if they seem to be at variance from commonly known facts and then decide which of the given conclusions logically follows from the given statements disregarding commonly known facts.

31. Statement :

Some penguins are peacocks.

Some peacocks are ducks.

Some ducks are swans.

Conclusion:

I. All ducks can never be penguins.

II. Some swans being peacocks is a possibility.

(A)  Only conclusion I is true

(B)  Both conclusion I and II are true

(C)  Either conclusion I or II is true

(D)  Only conclusion II is true

(E)  Neither conclusion I nor II is true

Answer: (D)

32. Statements:

Some chocolates are pizzas.

All pizzas are burgers.

All burgers are ice-creams.

Conclusions:

I. Atleast some chocolates are burgers.

II. All ice-creams being chocolates is a possibility.

(A)  Only conclusion I is true    

(B)  Both conclusion I and II are true

(C)  Either conclusion I or II is true

(D)  Only conclusion II true

(E)  Neither conclusion I nor II is true

Answer: (B)

33. Statements:

No wall is floor.

All floors are roofs.

Some roots are windows.

Conclusions:

I. Some floors are definitely not windows.

II. Atleast some roofs are walls.

(A)  Only conclusion I is true    

(B)  Both conclusion I and II are true

(C)  Either conclusion I or II is true

(D)  Only conclusion II true

(E)  Neither conclusion I nor II is true

 Answer: (E)

34. Statements:

All plugs are wires.

Some wires are switches.

All switches are batteries.

Conclusions:

I. All plugs being switches is a possibility.

II. Atleast some batteries are plugs.

(A)  Only conclusion I is true    

(B)  Both conclusion I and II are true

(C)  Either conclusion I or II is true

(D)  Only conclusion II true

(E)  Neither conclusion I nor II is true

Answer: (A)

35. Statements:

Some cakes are biscuits.

All biscuits are pastries.

Some pastries are doughnuts.

Conclusions:

I. All doughnuts being cakes is a possibility.

II. Atleast some cakes are pastries.

(A)  Only conclusion I is true    

(B)  Both conclusion I and II are true

(C)  Either conclusion I or II is true

(D)  Only conclusion II true

(E)  Neither conclusion I nor II is true

Answer: (B)

36. Statements:

No green is yellow.

All yellow are red.

All red are blue.

Conclusions:

I. All blues being greens is a possibility.

II. Atleast some blues are yellows.

(A)  Only conclusion I is true    

(B)  Both conclusion I and II are true

(C)  Either conclusion I or II is true

(D)  Only conclusion II true

(E)  Neither conclusion I nor II is true

Answer: (D)

Directions – (Q. 37-40) In this question, relationship between different elements is shown in the statements. The statement is followed by conclusions. Study the conclusions based on the given statement and select the appropriate answer.

37. Statements: G < A ≤ M = E ≥ S; Q ≥ E < O; D ≤

Conclusions: I. A ≤ Q.

                        II. G < S.

(A)  Only conclusion I is true

(B)  Both conclusion I and II are true

(C)  Either conclusion I or II is true

(D)  Only conclusion II is true

(E)  Neither conclusion I nor II is true

Answer: (A)

38. Statements: A ≤ B = C ≤D ≤ E > F; X≤ D > Z.

Conclusions: I. A ≥ Z.

                        II. X < F.

(A)  Only conclusion I is true

(B)  Both conclusion I and II are true

(C)  Either conclusion I or II is true

(D)  Only conclusion II is true

(E)  Neither conclusion I nor II is true

Answer: (E)

39. Statements: G < A ≤ M = E ≥ S; Q ≥ E < O; D ≤

Conclusion: I. E ≥ D.

                     II. M < O.

(A)  Only conclusion I is true

(B)  Both conclusion I and II are true

(C)  Either conclusion I or II is true

(D)  Only conclusion II is true

(E)  Neither conclusion I nor II is true

Answer: (B)

40. Statements: L = A ≤ M > P; A ≥ J = S < R.

Conclusions I. M ≥ J.

                      II. R > L.

(A)  Only conclusion I is true

(B)  Both conclusion I and II are true

(C)  Either conclusion I or II is true

(D)  Only conclusion II is true

(E)  Neither conclusion I nor II is true

Answer: (A)

Directions- (Q. 41-45) This questions consists of a question and two statements numbered I and II given below it. You have to decide whether the data given in the statements are sufficient to answer the questions. Read both the statements and choose the most appropriate option.

41. Is P the husband of Q?

I. S is the brother of Q. V is the only brother of S. M is the father of V. M has only three children. P is the son-in-law of M.

II. P is married to Q. A is the mother-in-law of Q. A is the mother of E. G is the only sister of E.

(A)  The data in statement II alone is sufficient to answer the question while the data in statement I alone is not sufficient to answer the question.

(B)  The data in both statements I and II together are necessary to answer the question.

(C)  The data in statement I alone is sufficient to answer the question while data in statement II alone is not sufficient to answer the question.

(D)  The data either in statement I alone or statement II alone are sufficient to answer the question.

(E)  The data even in both the statement I and II together are not sufficient to answer the question.

Answer: (C)

42. How is ‘import’ definitely coded in the given code language? (All the given codes are two letter codes only)

I. ‘import high quality goods’ is coded as ‘fa ju ra ti’ and ‘monitor quality of goods’ is written as ‘mp fa cd ju’.

II. ‘import uranium for plant’ is coded as ‘na ti vo ku’ and ‘plant for uranium access’ is coded as ‘vo zy ku na’.

(A)  The data in statement II alone is sufficient to answer the question while the data in statement I alone is not sufficient to answer the question.

(B)  The data in both statements I and II together are necessary to answer the question.

(C)  The data in statement I alone is sufficient to answer the question while data in statement II alone is not sufficient to answer the question.

(D)  The data either in statement I alone or statement II alone are sufficient to answer the question.

(E)  The data even in both the statement I and II together are not sufficient to answer the question.

Answer: (A)

43. In a five storey building (consisting of floors numbered 1 to 5, wherein the ground floor is numbered 1, the floor above it is numbered and so on till the topmost floor is numbered 5.) each of the five friends namely A, B, C, D and E lives on a different floor. On which floor number does D live?

I. C lives on an even numbered floor. Only two person live between C and E. B lives on an even numbered floor. A lives immediately below B.

II. A lives on an odd numbered floor but not on the lowermost floor. E lives on one of the floors above A. Only two people live between E and C. No one lives between C and D.

(A)  The data in statement II alone is sufficient to answer the question while the data in statement I alone is not sufficient to answer the question.

(B)  The data in both statements I and II together are necessary to answer the question.

(C)  The data in statement I alone is sufficient to answer the question while data in statement II alone is not sufficient to answer the question.

(D)  The data either in statement I alone or statement II alone are sufficient to answer the question.

(E)  The data even in both the statement I and II together are not sufficient to answer the question.

Answer: (C)

44. Five different exams Q, R, S, T and U were conducted in January, February, July, September and December of the same year (but not necessarily in the same order). In which month was exam Q conducted? (No exam was conducted in any other month in the year).

I. U was conducted in a month having less than 31 days. Only two exams were conducted between U and T. No exam was conducted between Q and S.

II. R was conducted in a month having 31 days but not July. Only three exams were held between R and T. No exams were held between U and S.

(A)  The data in statement II alone is sufficient to answer the question while the data in statement I alone is not sufficient to answer the question.

(B)  The data in both statements I and II together are necessary to answer the question.

(C)  The data in statement I alone is sufficient to answer the question while data in statement II alone is not sufficient to answer the question.

(D)  The data either in statement I alone or statement II alone are sufficient to answer the question.

(E)  The data even in both the statement I and II together are not sufficient to answer the question.

Answer: (E)

45. Three boxes, A, B and C, each having a different colour, yellow, blue and red are kept one above the other (but not necessarily in the same order). How many boxes (none/one/two) are kept below the blue covered box?

I. Only one box is kept between the blue and the yellow box. A is kept immediately below the red box.

II. Yellow coloured box is the lowermost. Blue box is kept immediately above C. B is not blue in colour.

(A)  The data in statement II alone is sufficient to answer the question while the data in statement I alone is not sufficient to answer the question.

(B)  The data in both statements I and II together are necessary to answer the question.

(C)  The data in statement I alone is sufficient to answer the question while data in statement II alone is not sufficient to answer the question.

(D)  The data either in statement I alone or statement II alone are sufficient to answer the question.

(E)  The data even in both the statement I and II together are not sufficient to answer the question.

Answer: (Cannot be determined)

Directions- (Q. 46-48) Study the following information and answer question-

    Six friends Sunny, Saksham, Rishi, Rupali, Preeti and Sandeep won different number of chocolates at an event on the same day.

• Rishi won less chocolates than Sandeep but more than Rupali and Preeti.

• Rishi won less chocolates than only two persons.

• Pretti won less chocolates than Rupali but more than Sunny. Sandeep didn’t win the maximum number of chocolates.

• The one who won second highest number of chocolates won 41 chocolates. Preeti won 25 chocholates.

46. How many chocolates did Saksham probably win ?

(A)  44

(B)  41

(C)  20

(D)  40

(E)  38

Answer: (A)

47. Which of the following possibly won 20 chocolates?

(A)  Rishi

(B)  Saksham

(C)  Sandeep

(D)  Sunny

(E)  Rupali

Answer: (D)

48. If Rishi won 32 chocoaltes, then how many chocolates did Rupali probably win?

(A)  23

(B)  27

(C)  21

(D)  33

(E)  37

Answer: (B)

49. Read he given information and answer the question-

Continuous use of sanitizers should be avoided among young children as they ingest a certain quantity of sanitizer with very use which can increase the percent content of chemical A in their body to as high as 0.04%.

Which of the following can be concluded from the given statement ?

(a)   Chemical A is either present in sanitizers or in formed in the body with the help of an agent present in the sanitizers.

(b)   Adults and older children are not affected by the sanitizers if ingested in small quantities.

(c)   Percentage of chemical A in the body under normal conditions is zero.

(d)   0.03% of chemical A in the body does not lead to any pathological conditions.

(A)  Only (c)

(B)  Both (b) and (d)

(C)  Only (a)

(D)  Both (a) and (c)

(E)  Both (a) and (d)

Answer: (B)

50. If it is possible to make only one meaningful English word with the first, second, third and fifth letters of the word ‘TRANSFORM’, which would be the second letter of that word from the left end ? If more than one such word can be formed, give ‘X’ as your answer. If no such word can be formed, give, ‘Z’ as your answer.

(A)  T

(B)  X

(C)  Z

(D)  A

(E)  R

Answer: (B)

© Copyright Entrance India - Engineering and Medical Entrance Exams in India | Website Maintained by Firewall Firm - IT Monteur